Shoulder & Elbow Module July 22 Flashcards

1
Q

A 12-year-old boy who pitches on two “select” baseball teams has had pain in his dominant right shoulder for the past 6 weeks. The pain is present only with throwing and is associated with decreased throwing velocity and control. He has no radiation of pain or paraesthesias of the upper extremity. An AP radiograph and MRI scan are shown in Figures 19a and 19b, respectively - widening proximal humerus physis

Management should consist of:

rest from throwing activities.
a subacromial corticosteroid injection.
open reduction and internal fixation.
arthroscopic labral repair.
biopsy of the proximal humerus.
A

Rest from throwing activities.

The imaging study demonstrates characteristics of Little Leaguer’s shoulder, including physeal widening. This condition is secondary to overuse (typically throwing) and responds well to rest from the inciting activity. There is no evidence from the patient’s history or examination that he has an impingement syndrome, nor is there any indication of labral pathology on the MRI scan. The changes in the proximal humerus are classic for this condition and are not suggestive of a neoplastic process requiring biopsy for definitive diagnosis.

Little Leaguer’s shoulder is an overuse injury occuring in young baseball pitchers resulting in epiphysiolysis of the proximal humerus (a Salter Harris Type 1 injury).
Diagnosis is made with radiographs of the shoulder showing a widened proximal humerus physis in comparison to contralateral shoulder.
Treatment is cessation of throwing, followed by PT and progressive throwing program after sufficient rest.

M>F
11-16 years - skeletally immature overhead athletes
10% all shoulder pain in paediatrics is related to throwing.

Mechanism: repetitive torsional and distractive stresses at the physis (SHI injury) - affects hypertrophic zone of physis (weakest portion of the growth plate)

Pitching occurs in 3 phases:
- Late cocking: shoulder is maximally externally rotated, leading to extreme rotatory torque through the growth plate, approximately 400% greater than the fragile physeal cartilage can tolerate

  • Deceleration: opposing forces of forward arm motion and rotator cuff results in excessive eccentric physeal stress

Breaking pitches are implicated
Number of pitches is the most important factor

Presents with decreased pitch velocity and accuracy - causes diffuse arm and shoulder pain with throwing - worse in late cocking or deceleration phases.
Pain resolves with rest.

Examination: Point tenderness over lateral proximal humerus, at the level of the physis.
Pain reproduced with shoulder rotation, Glenohumeral internal rotation deficit.

How well did you know this?
1
Not at all
2
3
4
5
Perfectly
2
Q

During which phase of the overhead throwing cycle is a baseball pitcher most likely to rupture the medial ulnar collateral ligament complex of the elbow?

Follow-through
Ball release
Early acceleration
Early cocking
Wind-up
A

Early Acceleration

The medial UCL is subjected to near-failure tensile stresses during the late cocking/early acceleration phase of throwing.

The medial ulnar collateral ligament, or medial collateral ligament of the elbow, is composed of three bundles:

  • an anterior bundle
  • a posterior bundle
  • a variable transverse oblique bundle.

The anterior bundle of the ulnar collateral ligament is the primary restraint to valgus force of the elbow from 30 to 120 degrees of flexion.

Biomechanical testing has shown that valgus forces as high as 64 N.m at the elbow during late cocking and early acceleration phases of throwing with compressive forces of 500 N at the lateral radiocapitellar articulation as the elbow moves from 110 to 20 degrees of flexion and velocities as high as 3000 deg/sec.

Mechanisms of injury MUCL:
- Acute trauma: often associated with elbow dislocations
- Overuse injuries:
+ Microtrauma from repetitive valgus stress leads to rupture of anterior band of MUCL
+ Baseball pitchers place significant valgus stress on the elbow in the late cocking and early acceleration phase of throwing
+ elbow valgus load increases with poor throwing mechanics and decreases with trunk-scapular kinesis, forearm pronation, dynamic flexor-pronator stabilization

  • Iatrogenic : excessive olecranon osteophyte resection places MCL at risk
How well did you know this?
1
Not at all
2
3
4
5
Perfectly
3
Q

For Grade III AC joint separations, surgical treatment results in which of the following when compared to non-operative management?

Faster return to play
Increased range of motion
Increased functional rotator cuff strength
Decreased funtional rotator cuff strength
Higher complication rate

A

Higher complication rate

Treatment of grade III AC separations remains somewhat controversial. A recent systematic review by Spencer concluded that the results of surgical treatment were not clearly any better than non-operative, had a higher complication rate, and a longer recovery prior to return to sport/work.

An acromioclavicular joint injury, otherwise known as a shoulder separation, is a traumatic injury to the acromioclavicular (AC) joint with disruption of the acromioclavicular ligaments and/or coracoclavicular (CC) ligaments.
Diagnosis is made with bilateral focused shoulder radiographs to assess for AC and CC interval widening.
Treatment is immobilzation or surgical reconstruction depending on patient activity levels, degree of separation and degree of ligament injury.

More common in male athletes - common injury making up 9% of shoulder girdle injuries.
Often sustained by falling onto the shoulder or a direct blow to the shoulder.

ACJ: diarthrodial joints composed of the articulation between the scapula (medial acromion) and the lateral clavicle with fibrocartilaginous intra-articular disc (involutes with age and tends to disintegrate by the age of 40).
Joint surface has an oblique orientation.
Motion: Primary gliding motion with minimal rotational motion (only 8deg rotation through ACJ due to synchronous scapuloclavicular motion).

Stability:
- Static:
+ Joint capsule
+ AC ligaments: controls horizontal motion and AP stability - S,I,A & P elements - with posterior and superior most important for stability.
+ Coracoclavicular ligaments: controls vertical motion and superior-inferior stability -
CONOID (medial - inserts on clavicle 4.5cm medial to the lateral edge, most important for vertical stability).
TRAPEZOID (lateral, inserts on clavicle 3cm medial to lateral edge)

  • Dynamic:
    Anterior deltoid and trapezius

Ex: Lateral clavicle/ACJ tenderness, abnormal contour of the shoulder compared to contralateral side.

ACJ exacerbation tests:

  • O’Brien’s Test: superficial pain localised to ACJ is suggestive of ACJ pathology, whereas deep pain is more suggestive of a SLAP lesion.
  • Crossbody adduction

Stability assessment:

  • Horizontal (anterior-posterior) stability evaluates AC ligaments: cross-body adduction, horizontal instability (ISAKOS type 3B) may indicate need for more aggressive treatment
  • Vertical (superior-inferior) stability evaluates CC ligaments

Rookwood Classification:
Type I: AC ligament sprain, no instability, reducible. Tx: sling

Type II: AC ligament torn, CC ligament sprain, AC horizontal instability. Reducible.
XR: ACJ disruption - Increased CC distance <25% vs contralateral.
Tx: sling.

Type III: Torn AC & CC ligaments. Reducible. Vertical Instability.
IIIA: No horizontal instability.
IIIB: horizontal instability.
XR: ACJ disruption - Increased CC distance <25%
Tx: Controversial.

Type IV: Torn AC & CC ligaments. Skin tenting and posterior fullness. Not reducible.
XR: Lateral clavicle displaced posterior through trapezius on the axillary lateral XR.
Tx: Surgical

Type V: Torn AC & CC ligaments. Not reducible. Severe shoulder droop, does not improve with shrug.
XR: ACJ disruption - Increased CC distance >100%
Tx: Surgical

Type VI: Torn AC & CC ligaments. Not reducible. Rare - associated injuries and paraesthesias more common.
XR: Inferior dislocation of lateral clavicle, lying either in subacromial or subcoracoid position
Tx: Surgical

Surgical techniques:
- Ligament reconstruction with soft tissue graft:
+ Modified Weaver-Dunn: distal clavicle excision with transfer of coracoacromial ligament to the distal clavicle to recreate CC ligament
+ autograft
+ allograft

  • Fixation: suture, hook plate, CC screw (Bosworth), cortical flip button (e.g Dog Bone)(+/- arthroscopic assistance), K-wire

Rehabilitation
Sling immobilization for 6 weeks, no shoulder range of motion
Return to full activity after 6 months

ORIF with CC screw fixation (Bosworth): fallen out of favour - screw placement from distal clavicle to coracoid, superior to inferior.
Pros: Rigid internal fixation
Cons: Danger of being too long and damaging critical structures below coracoid. Requires routine screw removal 8-12 weeks post op to prevent screw breakage (due to normal movement between clavicle and scapula.
Complicated often by hardware irritation and failure at level of screw purchase in the coracoid.

ORIF with CC suture fixation:
- Approach is proximal aspect of the anterolateral approach to the shoulder. Suture is placed either around or through the clavicle and around the base of the coracoid (can also use suture anchors for coracoid fixation).
Pros: No risk of hardware failure or migration.
Cons: Suture not as strong as screw fixation and requires careful suture passage inferior to coracoid due to proximity of crucial NV structures.
Complications: suture erosion causing distal third clavicle fracture, hardware irritation.

ORIF with AC hook plate:
- Expose distal and middle clavicle and use a standard hook plate over the superior distal clavicle.
Pros: rigid fixation.
Cons: hardware irritation - removal required if symptomatic.
Complications: acromial erosion, hook pullout

Phemister Technique (ORIF with AC pin fixation)
- Can be percutaneous - smooth wire or pin fixation directly across ACJ.
Cons: hardware irritation
Complications: high incidence of pin migration - limits use.

CC ligament reconstruction with coracoacromial (CA) ligament (Modified Weaver-Dunn)
- Open or arthroscopic distal clavicle excision and transfer of coracoacromial ligament to the distal clavicle to recreate CC ligament, then reinforce with internal fixation
Cons: coracoacromial ligament only 20% as strong as normal CC ligament, lack of internal fixation risks failure of soft tissue repair

CC ligament reconstruction with free tendon graft
approach
-Open or arthroscopically-assisted
grafts:
Autograft: palmaris longus, semitendinosus
Allograft: tibialis anterior

Figure-of-eight passage of graft, looping around coracoid and fixation through clavicular tunnels then reinforce with internal fixation.
Pros: graft reconstruction more closely recreates strength of native CC ligament
Cons: standard risks of allograft use or autograft harvest
lack of internal fixation risks failure of soft tissue repair

How well did you know this?
1
Not at all
2
3
4
5
Perfectly
4
Q

Posterior shoulder tightness can lead to a glenohumeral internal rotation deficit (GIRD). This has been linked most closely to which of the following shoulder pathologies?

Internal impingement
Humeral avulsion of the glenohumeral ligament
Subacromial impingement
Bicep tendinitis
Hill-Sachs lesion
A

Internal impingement

Repetitive overhead throwing can lead to posterior capsular stiffness and relative loss of internal rotation (GIRD). This may shift the contact point posterior and superior on the glenoid, leading to internal impingement where the greater tuberosity impinges on the posterosuperior labrum and posterior rotator cuff when the arm is abducted and externally rotated. Initial treatment involves posterior capsular stretching.

Myers et al evaluated two groups of throwing athletes, one with a diagnosis of internal impingement and one without, to compare the degree of GIRD/posterior capsular tightness and its correlation with increased external rotation gain. They found that throwing athletes with internal impingement demonstrated significantly greater GIRD and posterior shoulder tightness, and that management should include stretching to restore flexibility to the posterior shoulder.

Tyler et al sought to determine if improvements in GIRD and/or decreased posterior shoulder tightness were associated with a resolution of symptoms in 22 patients with internal impingement. After an average of 7 weeks of physical therapy, they found that resolution of symptoms was related to correction of posterior shoulder tightness but not correction of GIRD.

Glenohumeral internal rotation deficit (GIRD) is a condition resulting in the loss of internal rotation of the glenohumeral joint as compared to the contralateral shoulder, most commonly seen in the throwing athlete.
Diagnosis is made clinically with a decrease in internal rotation, increase in external rotation, with a decrease in total arc of rotation compared to the contralateral shoulder.
Treatment consists of physical therapy with a focus on posteroinferior capsular stretching.

Pathoanatomy: tightening of the posterior capsule or posteroinferior capsule leads to translation of humeral head (capsular restraint mechanism). Translation of humeral head is in the OPPOSITE direction from the area of capsular tightening. Posterior capsular leads to anterosuperior translation of the humeral head in flexion.
Posteroinferior capsular tightness leads to posterosuperior translation of the humeral head in ABER.
The anterior capsule is stretched.

Associated conditions:
- Glenohumeral instability

  • Internal impingement: abutment of the greater tuberosity against the posterosuperior glenoid during abduction and external rotation leads to pinching of posterosuperior rotator cuff
  • Articular-sided partial rotator cuff tears - due to tensile failure in excessive rotation and internal impingement
  • SLAP lesion: throwers with GIRD are 25% more likely to have a SLAP lesion. Peel-back mechanism (biceps anchor and postero superior labrum peels back) during late cocking because of posterosuperior translation of humeral head and change in biceps vector force posteriorly

On examination:
- Increased sulcus sign - due to stretching of anterior structures that resist external rotation (coracohumeral ligament, rotator interval).
- Characterised by altered glenohumeral ROM: decrease in IR and increase in ER.
When loss of IR is less than gain in ER, the shoulder retains normal kinematics, but when loss of IR exceeds gain in ER it leads to deranged kinematics.

Tx: Rest and physio - sleeper stretches to stretch posteroinferior capsule
Operative if extensive PT fails: posteroinferior capsule release or anterior stabilisation. Should immediately gain 65 deg internal rotation post op.

How well did you know this?
1
Not at all
2
3
4
5
Perfectly
5
Q

A 60-year-old patient fell down a flight of stairs and injured their right arm. Since the fall they are unable to move their extremity due to pain. Prior to the fall, the patient denied any pain in the shoulder or upper arm. Currently, the patient is neurovascularly intact. Figures A and B are the radiographs at the time of presentation. What is the best treatment option for this patient?

XR demonstrate well seated TSA with periprosthetic fracture at the level of the tip of the prosthesis.

Revision rTSA with cemented long-stem prosthesis
Revision rTSA with cementless long-stem prosthesis
ORIF with hybrid locking plate and cerclage cables
ORIF with lag screw fixation and neutralization plating
Nonoperative treatment

A

ORIF with hybrid locking plate and cerclage cables
The patient has sustained a Wright and Cofield type B periprosthetic humeral shaft fracture with a stable prosthesis. The best treatment option for this would involve ORIF with hybrid locking plate and cable construct.

Periprosthetic humeral shaft fractures are a relatively rare complication occurring in approximately 0.6-3% of patients that have undergone shoulder replacement procedures. These fractures pose treatment challenges as the prosthesis disrupts endosteal blood supply, causing higher nonunion rates.

Wright and Cofield system:
Type A fractures are proximal to the stem tip and are treated with ORIF;
Type B fractures are at the level of the stem tip and are treated with ORIF;
Type C fractures are distal to the stem tip and can be initially treated nonoperatively.

Steinmann et al. reviewed the treatment of periprosthetic humeral shaft fractures.

For intraoperative fractures, the authors recommended placement of a long stem prosthesis that bypasses the fracture site by at least 2 cortical diameters.
For postoperative type A and B fractures, treatment depends on whether the stem is loose or well fixed. Loose prostheses necessitate revision long stem component with supplementary fixation, whereas well-fixed stems require hybrid plate fixation. Type C fractures can be treated non-operatively, but in the presence of nonunion may require plate fixation with or without allograft struts.

High incidence of radial nerve palsy with distal humerus fractures - most often neuropraxia not requiring operative management.

Figures A and B are AP and lateral radiographs demonstrating a Wright and Cofield type B periprosthetic humeral shaft fracture. Illustration A depicts the Wright and Cofield classification system.

Incorrect Answers:
Answer 1: In the setting of a loose prosthesis, revision arthroplasty with a long stem implant would be the ideal choice. Cement fixation can be utilized in the presence of osteoporotic bone, but care must be taken to prevent cement extrusion into the fracture site. In this case, the stem is well fixed and revision arthroplasty is unnecessary.
Answer 2: Revision arthroplasty is indicated in a loose stem, but in this case, the stem is well fixed.
Answer 4: Lag screw fixation is not recommended with these fractures. Current data suggest good outcomes with hybrid locking plates and cerclage cables.
Answer 5: Type C fractures may be amenable to nonoperative treatment. However, when there is involvement near the stem tip there is a high risk of nonunion without adequate fixation due to poor endosteal blood flow in this region.

How well did you know this?
1
Not at all
2
3
4
5
Perfectly
6
Q

A 35-year-old carpenter has pain in the antecubital fossa that is worse with turning a screwdriver. He has undergone non-operative treatment for 6 months without relief. On physical examination his hook test is normal and there is pain and weakness with resisted supination. Radiographs are shown in Figures A-C. A MRI of the right elbow is shown in Figure D. The next most appropriate treatment is?

Figures A-C are normal radiographs of the elbow. Figure D is a cross-referenced axial and coronal T2 MRI that demonstrates increased signal around distal biceps insertion.

Exploration of the radial tunnel
Superficial radial neurectomy
Detachment and repair of the biceps tendon
Transfer of the biceps to the brachialis
EMG with nerve conduction study
A

Detachment and repair of the biceps tendon

While complete traumatic rupture of the distal biceps is more common, partial tears have been reported in the literature. The most common presentation is pain in the antecubital fossa worse with resisted supination.

Conservative management consists of NSAID’s, splinting and physical therapy. The distal biceps hook test is helpful in detecting full thickness distal biceps tears but not partial tears.

In one study by Vardakas et al, 7 partial distal biceps ruptures were treated with surgical debridement and reattachment with all patients reporting a significant decrease in their pain. Transfer to the brachialis improves flexion strength but not supination.

Ramsey et al present a review article on distal biceps tendon injuries. They state that the most successful management of partial distal biceps tears that have failed conservative management is to surgically treat it like a complete rupture with release and surgical reattachment of the distal biceps to the radial tuberosity.

Distal Biceps Avulsions are injuries to the biceps tendon at the radial tuberosity insertion that generally occurs due to a sudden excessive eccentric contraction of the biceps brachii.
Diagnosis can be made clinically in the setting of complete tears with a hook test. MRI studies can be used to discern between a complete tear and a partial tear.
Treatment can be nonoperative or operative depending on patient age, patient activity demands, chronicity of tear, and degree of tear.

Contents of antecubital fossa (medial to lateral):

  • Median nerve -> brachial artery -> biceps tendon -> radial nerve
  • Radial recurrent vessels lie superficial to biceps tendon

Distal biceps tendon possesses two distinct insertions:

  • Short head attaches distally on radial tuberosity (thin sliver), origin is coracoid processs, is a better flexor
  • Long head attaches proximally on radial tuberosity (oval footprint), origin is the superior lip of the glenoid and glenoid labrum, is a better supinator as attachment is furthest from axis of rotation (attaches to apex of radial tuberosity). Independent function to prevent anterior, inferior and superior translation of humeral head against proximal pull of short head of biceps

Lacertus fibrosus
Distal to the elbow crease, the biceps tendon gives off, from its medial side, the lacertus fibrosus (bicipital aponeurosis or biceps fascia).
Originates from the distal short head of the biceps tendon. Lacertus passes obliquely across the cubital fossa, running distally and medially, helping to protect the underlying brachial artery and median nerve. It is continuous with the deep fascia of the flexor tendon origin, envelopes flexor muscle bellies - may be mistaken for an intact distal biceps tendon on clinical exam.

Patient often experiences a painful “pop” as the elbow is eccentrically loaded from flexion to extension.

Ex: varying degree of proximal retraction of the muscle belly (“reverse Popeye sign”), medial ecchymosis, palpable defect.
Loss of more supination than flexion strength (loss 30% flexion, 40% supination, 50% sustained supination)

Hook test - ask patient to actively flex elbow to 90 and to fully supinate forearm - examiner then attempts to hook the lateral edge of the biceps tendon with index finger - if intact/partially torn then finger can be inserted 1cm beneath the tendon.

False positive: partial tear, intact lacertus fibrosis, underlying brachialis tendon

Ruland Biceps squeeze (akin to Simmonds for Achilles) - elbow held in 60-80° of flexion with the forearm slightly pronated. One hand stabilizes the elbow while the other hand squeezes across the distal biceps muscle belly. A positive test is failure to observe supination of the patient’s forearm or wrist.

Subacute/chronic ruptures may be treated successfully with direct repair (without allograft) - may need to hyperflex elbow to achieve fixation. Hyperflexion does NOT lead to loss of elbow ROM or flexion contracture
timing.
Surgical treatment should occur within a few weeks from the date of injury - further delay may preclude a straightforward, primary repair.
A more extensile approach may be required in a chronic rupture to retrieve the retracted and scarred distal biceps tendon.

Limited antecubital fossa incision:
- Interval between the brachioradialis and pronator teres - radial (lateral) retraction of the brachioradialis and medial retraction of the pronator teres.
Lateral antebrachial cutaneous nerve (LABCN) is identified as it exits between the biceps and brachialis at antecubital fossa.
Recurrent radial vessels encountered and either coagulated or carefully dissected and retracted.
Protect PIN by limiting forceful lateral retraction and maintaining supination

Cortical button - tendon end is whip-stitched with the suture ends placed into two central holes of the button.
Acorn reamer is used to ream through the anterior cortex after exposing tuberosity.
A smaller hole is then drilled through the far cortex to allow the button to be passed across the far cortex.
Button is flipped to lie on far cortex, and suture ends are tensioned (tension slide) to bring tendon into tunnel

Complications: injury to the LABCN is most common whereas radial nerve or PIN injury is most severe - risk has decreased with new tendon fixation techniques that require less dissection in the antecubital fossa
Synostosis and resulting loss of pronation/supination: avoid exposing periosteum of ulna & avoid dissection between the radius and ulna
Heterotopic ossification

Postoperative: immobilize in 110° of flexion and moderate supination

How well did you know this?
1
Not at all
2
3
4
5
Perfectly
7
Q

A 46-year-old woman fell from her bicycle and sustained the injury shown in Figure 24. Which of the following ligaments has been disrupted?

XR: Increased CC distance > 100% of contralateral

Acromioclavicular
Acromioclavicular and coracoclavicular
Coracoclavicular
Coracoacromial and sternoclavicular
Sternoclavicular
A

Acromioclavicular and coracoclavicular

The radiograph shows a type V acromioclavicular joint injury. Type V injuries involve disruption of the acromioclavicular and coracoclavicular ligaments. Type I injuries involve a sprain of the acromioclavicular joint ligaments. Type II injuries involve disruption of the acromioclavicular joint ligaments; the coracoclavicular ligaments are partially injured. Sternoclavicular ligaments stabilize the medial clavicle and the sternum; they are not damaged with acromioclavicular joint dislocations.

An acromioclavicular joint injury, otherwise known as a shoulder separation, is a traumatic injury to the acromioclavicular (AC) joint with disruption of the acromioclavicular ligaments and/or coracoclavicular (CC) ligaments.
Diagnosis is made with bilateral focused shoulder radiographs to assess for AC and CC interval widening.
Treatment is immobilzation or surgical reconstruction depending on patient activity levels, degree of separation and degree of ligament injury.

More common in male athletes - common injury making up 9% of shoulder girdle injuries.
Often sustained by falling onto the shoulder or a direct blow to the shoulder.

ACJ: diarthrodial joints composed of the articulation between the scapula (medial acromion) and the lateral clavicle with fibrocartilaginous intra-articular disc (involutes with age and tends to disintegrate by the age of 40).
Joint surface has an oblique orientation.
Motion: Primary gliding motion with minimal rotational motion (only 8deg rotation through ACJ due to synchronous scapuloclavicular motion).

Stability:
- Static:
+ Joint capsule
+ AC ligaments: controls horizontal motion and AP stability - S,I,A & P elements - with posterior and superior most important for stability.
+ Coracoclavicular ligaments: controls vertical motion and superior-inferior stability -
CONOID (medial - inserts on clavicle 4.5cm medial to the lateral edge, most important for vertical stability).
TRAPEZOID (lateral, inserts on clavicle 3cm medial to lateral edge)

  • Dynamic:
    Anterior deltoid and trapezius

Ex: Lateral clavicle/ACJ tenderness, abnormal contour of the shoulder compared to contralateral side.

ACJ exacerbation tests:

  • O’Brien’s Test: superficial pain localised to ACJ is suggestive of ACJ pathology, whereas deep pain is more suggestive of a SLAP lesion.
  • Crossbody adduction

Stability assessment:

  • Horizontal (anterior-posterior) stability evaluates AC ligaments: cross-body adduction, horizontal instability (ISAKOS type 3B) may indicate need for more aggressive treatment
  • Vertical (superior-inferior) stability evaluates CC ligaments

Rookwood Classification:
Type I: AC ligament sprain, no instability, reducible. Tx: sling

Type II: AC ligament torn, CC ligament sprain, AC horizontal instability. Reducible.
XR: ACJ disruption - Increased CC distance <25% vs contralateral.
Tx: sling.

Type III: Torn AC & CC ligaments. Reducible. Vertical Instability.
IIIA: No horizontal instability.
IIIB: horizontal instability.
XR: ACJ disruption - Increased CC distance <25%
Tx: Controversial.

Type IV: Torn AC & CC ligaments. Skin tenting and posterior fullness. Not reducible.
XR: Lateral clavicle displaced posterior through trapezius on the axillary lateral XR.
Tx: Surgical

Type V: Torn AC & CC ligaments. Not reducible. Severe shoulder droop, does not improve with shrug.
XR: ACJ disruption - Increased CC distance >100%
Tx: Surgical

Type VI: Torn AC & CC ligaments. Not reducible. Rare - associated injuries and paraesthesias more common.
XR: Inferior dislocation of lateral clavicle, lying either in subacromial or subcoracoid position
Tx: Surgical

Surgical techniques:
- Ligament reconstruction with soft tissue graft:
+ Modified Weaver-Dunn: distal clavicle excision with transfer of coracoacromial ligament to the distal clavicle to recreate CC ligament
+ autograft
+ allograft

  • Fixation: suture, hook plate, CC screw (Bosworth), cortical flip button (e.g Dog Bone)(+/- arthroscopic assistance), K-wire

Rehabilitation
Sling immobilization for 6 weeks, no shoulder range of motion
Return to full activity after 6 months

ORIF with CC screw fixation (Bosworth): fallen out of favour - screw placement from distal clavicle to coracoid, superior to inferior.
Pros: Rigid internal fixation
Cons: Danger of being too long and damaging critical structures below coracoid. Requires routine screw removal 8-12 weeks post op to prevent screw breakage (due to normal movement between clavicle and scapula.
Complicated often by hardware irritation and failure at level of screw purchase in the coracoid.

ORIF with CC suture fixation:
- Approach is proximal aspect of the anterolateral approach to the shoulder. Suture is placed either around or through the clavicle and around the base of the coracoid (can also use suture anchors for coracoid fixation).
Pros: No risk of hardware failure or migration.
Cons: Suture not as strong as screw fixation and requires careful suture passage inferior to coracoid due to proximity of crucial NV structures.
Complications: suture erosion causing distal third clavicle fracture, hardware irritation.

ORIF with AC hook plate:
- Expose distal and middle clavicle and use a standard hook plate over the superior distal clavicle.
Pros: rigid fixation.
Cons: hardware irritation - removal required if symptomatic.
Complications: acromial erosion, hook pullout

Phemister Technique (ORIF with AC pin fixation)
- Can be percutaneous - smooth wire or pin fixation directly across ACJ.
Cons: hardware irritation
Complications: high incidence of pin migration - limits use.

CC ligament reconstruction with coracoacromial (CA) ligament (Modified Weaver-Dunn)
- Open or arthroscopic distal clavicle excision and transfer of coracoacromial ligament to the distal clavicle to recreate CC ligament, then reinforce with internal fixation
Cons: coracoacromial ligament only 20% as strong as normal CC ligament, lack of internal fixation risks failure of soft tissue repair

CC ligament reconstruction with free tendon graft
approach
-Open or arthroscopically-assisted
grafts:
Autograft: palmaris longus, semitendinosus
Allograft: tibialis anterior

Figure-of-eight passage of graft, looping around coracoid and fixation through clavicular tunnels then reinforce with internal fixation.
Pros: graft reconstruction more closely recreates strength of native CC ligament
Cons: standard risks of allograft use or autograft harvest
lack of internal fixation risks failure of soft tissue repair

How well did you know this?
1
Not at all
2
3
4
5
Perfectly
8
Q

A 22-year-old volleyball player reports the insidious onset of superior and posterior shoulder pain. Radiographs are normal. An MRI scan is shown in Figure 25. What is the most specific physical examination finding?

MRI: spinoglenoid cyst

Positive impingement sign
Positive apprehension
Positive active compression
Weakness of external rotation
Weakness of abduction
A

Weakness of external rotation

Overhead athletes are prone to a number of problems involving the shoulder. Pitchers and volleyball players are susceptible to posterior superior labral tears and internal impingement. These patients will have posterior superior shoulder pain, a positive relocation sign, and a positive active compression test. Occasionally, these posterior superior labral tears are associated with a spinoglenoid cyst as seen in the MRI scan. These cysts cause compression of the suprascapular nerve which manifests primarily as weakness of the infraspinatus, resulting in weakness of external rotation.

Suprascapular neuropathy is compression of the suprascapular nerve that most commonly occurs at the suprascapular notch or spinoglenoid notch by a mass (i.e cyst).
Diagnosis can be suspected clincally with weakness and atrophy of the infraspinatous or supraspinatous and confirmed with MRI studies showing cysts in the suprascapular notch or spinoglenoid notch.
Treatment of suprascapular nerve compression at the suprascapular notch requires decompression of a cyst when present. Treatment of a spinoglenoid cyst requires either decompression or repair of an associated labral lesion (if present).

Suprascapular notch entrapment:
Proximal compression of supra scapular nerve in the supra scapular notch leads to weakness of both supraspinatus and infraspinatus. Suprascapular nerve (C5,6) - emerges off superior trunk of brachial plexus, travels across posterior triangle of neck to scapula innervation of supraspinatus and infraspinatus. Compression can be due to a ganglion cyst (often associated with labral tears), transverse scapular ligament entrapment, fracture callus.

Spinoglenoid notch entrapment:
Distal compression of supra scapular nerve therefore weakness of infraspinatus only. Compression can be due to posterior labral tears -> cyst, spinoglenoid ligament, spinoglenoid notch ganglion, traction injury (volleyball players), transglenoid fixation

Suprascapular ligament arises from medial base of coracoid and overlies suprascapular notch - suprascapular artery runs above and suprascapular nerve runs below.

Spinoglenoid ligament arises near spinoglenoid notch and overlies distal suprascapular nerve

How well did you know this?
1
Not at all
2
3
4
5
Perfectly
9
Q

What is the average medial-to-lateral distance of the supraspinatus tendon insertion at its footprint on the greater tuberosity?

6-8mm
14-16mm
20-22mm
24-26mm
30-32mm
A

Cadaveric studies have shown the average medial-to-lateral distance of the supraspinatus tendon footprint on the greater tuberosity is 14-16mm.

How well did you know this?
1
Not at all
2
3
4
5
Perfectly
10
Q

A 28-year-old professional baseball pitcher sustains a complete rupture of his ulnar collateral ligament. He is neurovascularly intact on exam. Which of the following surgical reconstruction techniques has been shown to result in the lowest complication rate and best patient outcome?

Splitting of flexor-pronator mass, figure-of-8 graft fixation.
Splitting of flexor-pronator mass, docking graft fixation.
Splitting of flexor-pronator mass, docking graft fixation, ulnar nerve transposition.
Detachment of flexor-pronator mass, figure-of-8 graft fixation, ulnar nerve transposition.
Detachment of flexor-pronator mass, docking graft fixation, ulnar nerve transposition.

A

Splitting of flexor-pronator mass, docking graft fixation.

Ulnar collateral ligament (UCL) reconstruction using a flexor-pronator muscle-splitting approach and a docking graft fixation technique are associated with the lowest complication rate and best patient outcomes.

Vitale et al. demonstrated that the flexor-pronator muscle-splitting approach was associated with better outcomes than detachment of the flexor-pronator mass, had a lower rate of postoperative ulnar neuropathy, and a lower overall complication rate. They also found fixation of the graft utilizing the docking technique was associated with better outcomes than the figure-of-8 technique. Abandoning the obligatory ulnar nerve transposition was associated with improved patient outcomes (89% vs. 75%) and a lower rate of postoperative ulnar neuropathy (4% vs. 9%).

Rettig et al performed a case series review of 31 overhead throwing athletes with ulnar collateral ligament injuries managed nonoperatively with 3 months rest followed by rehabilitation exercises. They concluded that 42% of athletes were able to return to their previous level of competition at an average of 6 months from diagnosis (earlier than reconstruction). The authors were unable to identify any patient-specific factors (duration of symptoms, age, acuity of onset) that would predict the success of nonoperative treatment.

In the docking graft fixation technique the graft is placed in a triangular configuration through a single humeral tunnel. The suture limbs are then brought out through two separate bone holes and tied over a bony bridge on the superior aspect of the medial epicondyle.

The figure-of-8 (Jobe) graft fixation technique is performed by passing the tendon graft through two bone tunnels in the medial epicondyle of the humerus and through one tunnel in the ulnar sublime tubercle. The graft is then sutured to itself in a figure-of-8 configuration.

Incorrect Answers:
Answer 1: The figure-of-8 technique is not associated with better patient outcomes when compared to the docking technique.
Answer 3: Obligatory ulnar nerve transposition during UCL reconstruction is associated with a higher rate postoperative ulnar neuropathy and worse patient outcomes, and therefore should be avoided.
Answers 4 and 5: Detachment of the flexor-pronator mass is not associated with better patient outcomes when compared to the muscle-splitting approach.

The medial ulnar collateral ligament, or medial collateral ligament of the elbow, is composed of three bundles:

  • an anterior bundle
  • a posterior bundle
  • a variable transverse oblique bundle.

The anterior bundle of the ulnar collateral ligament is the primary restraint to valgus force of the elbow from 30 to 120 degrees of flexion.

Biomechanical testing has shown that valgus forces as high as 64 N.m at the elbow during late cocking and early acceleration phases of throwing with compressive forces of 500 N at the lateral radiocapitellar articulation as the elbow moves from 110 to 20 degrees of flexion and velocities as high as 3000 deg/sec.

Mechanisms of injury MUCL:
- Acute trauma: often associated with elbow dislocations
- Overuse injuries:
+ Microtrauma from repetitive valgus stress leads to rupture of anterior band of MUCL
+ Baseball pitchers place significant valgus stress on the elbow in the late cocking and early acceleration phase of throwing
+ elbow valgus load increases with poor throwing mechanics and decreases with trunk-scapular kinesis, forearm pronation, dynamic flexor-pronator stabilization

  • Iatrogenic : excessive olecranon osteophyte resection places MCL at risk
How well did you know this?
1
Not at all
2
3
4
5
Perfectly
11
Q

A 25-year old female with a seizure disorder complains of persistent left shoulder pain after sustaining a seizure 1 week ago. She was placed in a sling in the ER and is following up in your office. Figure A shows the radiograph taken in the ER. On examination, her range of motion is limited and is only able to externally rotate to neutral. What is the next step in management?

Sling use for comfort and follow-up in 2 weeks
Repeat True AP radiograph
Axillary lateral radiograph
MRI of the shoulder
Intra-articular cortisone injection with range of motion exercises

A

Axillary lateral radiograph

This question tests the concept that posterior shoulder dislocation is frequently missed due to inadequate imaging. Trauma shoulder radiographs (which include an AP, axillary, and scapular Y view) must be obtained in all suspected shoulder dislocations.

Posterior dislocations are more common following a seizure. The posteriorly dislocated shoulder is typically held in IR and most consistent finding is a mechanical block to ER caused by the anterior humeral head defect on the posterior aspect of the glenoid.

According to the reference by Robinson et al, good functional outcomes are associated with early detection and treatment of isolated posterior dislocations that are associated with a small osseous defect and are stable following closed reduction.

Posterior shoulder instability and dislocations are less common than anterior shoulder instability and dislocations, but are much more commonly missed.
Diagnosis is made radiographically in the setting of acute dislocations. Chronic instability can be diagnosed with presence of positive posterior instability provocative tests and confirmed with MRI studies showing posterior labral pathology.
Treatment may be nonoperative or operative depending on chronicity of symptoms, recurrence of instability, and the severity of labrum and/or glenoid defects.

Risk factors for posterior dislocation include bony abnormality (glenoid retroversion or hypoplasia) and ligamentous laxity.

Mechanism:
Trauma - posterior dislocation - usually significant trauma.
Microtrauma - posterior instability - can lead to labral tear, incomplete labral avulsion or erosion of the posterior labrum which may lead to gradual stretching of the capsule and patulous posterior capsule - usually insidious onset and presentation.
Seizures and electric shock - tetanic muscles pull the humeral head out

Biomechanical forces: flexed, adducted and IR rotated arm = high risk position

Static restraint: labrum deepens glenoid by 50%

Primary stabilisers of posterior shoulder:

  • Posterior band of IGHL: primary restraint in internal rotation
  • Subscapularis: primary dynamic restraint in external rotation and against posterior subluxation
  • Superior glenohumeral ligament and coracohumeral ligament: primary restraint to inferior translation of the adducted arm and to external rotation and primary static stabilizer to posterior subluxation with shoulder in flexion, adduction, and internal rotation.

Acute posterior dislocation -> limited external rotation, with the arm locked in an IR position. Pain on flexion, adduction and IR for posterior instability.

Provocative tests:

  • Jerk test
  • Kim test
  • Posterior stress test
  • Posterior load and shift test
How well did you know this?
1
Not at all
2
3
4
5
Perfectly
12
Q

A 62-year-old man complains of shoulder pain for 2 years. He has had 1 course of intra-articular sodium hyaluronate and 6 weeks of physical therapy with little relief. Examination reveals diminished arm flexion and abduction secondary to pain. Radiographs of his shoulder are shown in Figures A and B. According to the American Academy of Orthopaedic Surgeons Clinical Practice Guidelines, what is the next best step?

XR: end stage GHJ OA

Humeral head replacement arthroplasty
Hemiarthroplasty and ream-and-run glenoid procedure
Cuff tear arthropathy (CTA) prosthesis
Total shoulder arthroplasty with a metal-backed cemented glenoid component
Total shoulder arthroplasty with an all-polyethylene cemented glenoid component

A

Total shoulder arthroplasty with an all-polyethylene cemented glenoid component

This patient has end-stage glenohumeral osteoarthritis (GH OA). According to the AAOS CPG, total shoulder arthroplasty (TSA) is recommended using an all-polyethylene cemented glenoid component.

TSA is indicated for cases of end-stage GH OA. It is preferred to hemiarthroplasty. It is contraindicated in cases with insufficient glenoid bone stock (glenoid wear to the level of the coracoid), rotator cuff arthropathy or irreparable cuff tears and deltoid dysfunction. It provides good pain relief and has good survival at 10 years (>90%).

Figures A and B show end-stage GH OA with large osteophytes and subchondral sclerosis. There is significant glenoid wear and posterior subluxation (Walch B glenoid deformity).

Incorrect Answers:
Answer 1: The AAOS CPG does not recommend humeral head replacement arthroplasty (resurfacing).
Answer 2: Although the AAOS CPG recommends both hemiarthroplasty and TSA as options, TSA is preferred.
Answer 3: The AAOS CPG does not recommend use of a CTA humeral component.
Answer 4: The AAOS CPG does not recommend the use of metal-backed glenoid components. Metal-backed glenoids have higher rates of revision than all-polyethylene glenoids.

TSA:
Replacement of humeral head and glenoid resurfacing: cemented all-polyethylene glenoid resurfacing is standard of care

Total shoulder arthroplasty unique from THA and TKA in that:

  • Greater range of motion in the shoulder
  • Success depends on proper functioning of the soft tissues
  • Glenoid is less constrained: leads to greater sheer stresses and is more susceptible to mechanical loosening

Factors required for success of TSA
- Rotator cuff intact and functional: if rotator cuff is deficient and proximal migration of humerus is seen on x-rays (rotator cuff arthropathy) then glenoid resurfacing is contraindicated. If there is an irreparable rotator cuff deficiency then proceed with hemiarthroplasty or a reverse ball prosthesis. An isolated supraspinatus tear without retraction can proceed with TSA - incidence of full thickness rotator cuff tears in patients getting a TSA is 5% to 10% - if positive impingement signs on exam, order a pre-operative MRI.
- Glenoid bone stock and version
if glenoid is eroded down to coracoid process then glenoid resurfacing is contraindicated (Walch classification).

Outcomes
Pain relief more predictable than hemiarthroplasty, reliable range of motion, good survival at 10 years (93%), good longevity with cemented and press-fit humeral components, worse results for post-capsulorrhaphy arthropathy.

Indications

  • Pain (anterior to posterior), especially at night, and inability to perform activities of daily living
  • Glenoid chondral wear to bone: preferred over hemiarthroplasty for osteoarthritis and inflammatory arthritis
  • Posterior humeral head subluxation

Contraindications

  • Insufficient glenoid bone stock
  • Rotator cuff arthropathy
  • Deltoid dysfunction
  • Irreparable rotator cuff (hemiarthroplasty or reverse total shoulder are preferable) as risk of loosening of the glenoid prosthesis is high (“rocking horse” phenomenon)
  • Active infection
  • Brachial plexus palsy

Glenoid loosening - most common cause of TSA failure (30% of primary OA revisions)
Risk factors: insufficient glenoid bone stock (posterior glenoid wear associated with glenoid loosening), rotator cuff deficiency
2.9% reoperation rate for loosening (28% with revision)

Vascular injury
Arcuate artery, branch off the anterior humeral circumflex artery, can be damaged during biceps tendon elevation

Humeral stem loosening: more common in RA and osteonecrosis. Rule out infection.

Subscapularis repair failure

Malposition of components

Improper soft tissue balancing: failure due to undiagnosed presence of rotator cuff tears

Iatrogenic rotator cuff injury can occur if humeral neck osteotomy is inferior to level of rotator cuff insertion. Overstuffing glenohumeral joint leading to attritional supraspinatus and subscapularis tears

Stiffness

Infection: may have normal aspiration results
culture. Infection rate 1-2% after primary TSA.
Arthroscopic tissue culture more sensitive (100% sensitive and specific) than fluoroscopically guided aspiration (17% sensitivity, 100% specific)
Propionibacterium acnes (P. acnes),
now referred to as Cutibacterium acnes (c. acnes) most common cause of indolent infections and implant failures - gram positive, facultative, aerotolerant, anaerobic rod that ferments lactose to propionic acid
has high bacterial burden around the shoulder forms biofilm.
P. acnes PJI more common in males
Use anaerobic culture bottles, keep for 10-14days (mean time to detection 6 days)

Neurologic injury: axillary nerve is most commonly injured, musculocutaneous nerve can be injured by retractor placement under conjoint tendon

Periprosthetic fracture: acceptable fragment alignment ≤ 20° flexion/extension, ≤ 30° varus/valgus, ≤ 20° rotation malalignment

Wright and Cofield system:
Type A fractures are proximal to the stem tip and are treated with ORIF;
Type B fractures are at the level of the stem tip and are treated with ORIF;
Type C fractures are distal to the stem tip and can be initially treated nonoperatively.

Steinmann et al. reviewed the treatment of periprosthetic humeral shaft fractures.

For intraoperative fractures, the authors recommended placement of a long stem prosthesis that bypasses the fracture site by at least 2 cortical diameters.
For postoperative type A and B fractures, treatment depends on whether the stem is loose or well fixed. Loose prostheses necessitate revision long stem component with supplementary fixation, whereas well-fixed stems require hybrid plate fixation. Type C fractures can be treated non-operatively, but in the presence of nonunion may require plate fixation with or without allograft struts.

How well did you know this?
1
Not at all
2
3
4
5
Perfectly
13
Q

During a total shoulder arthoplasty (TSA), which of the following technical maneuvers would most likely place the rotator cuff tendons at risk of injury?

Excessive retraction on the deltoid muscle during a delto-pectoral approach
Palpation of the rotator cuff insertion prior to humeral head resection
A humeral cut with 30 degrees of retroversion
Excessive bone removal with the humeral neck osteotomy
A humeral cut with 45 degrees of inclination

A

Excessive bone removal with the humeral neck osteotomy

The rotator cuff tendons can be inadvertantly cut or detached during a TSA if the head cut is made either too distally or in excessive retroversion.

Pearl et al studied the placement of humeral component position during TSA by studying 21 cadaveric specimens they recommend anatomic reconstruction of the retroversion angle based on patient anatomy. They also stress palpation of the rotator cuff insertion prior to humeral head resection to avoid inadvertant cuff injury.

Appropriate osteotomy is made proximal to both the greater and less tuberosities and medial to SIT insertion.

Incorrect Answers:
Choice 1- Excessive retraction on the deltoid muscle could cause injury to the axillary nerve, but will not injure the rotator cuff.
Choices 2- This step is encouraged to spare the rotator cuff insertions before humeral head osteotomy
Choice 3- A head cut in 30 degrees of retroversion is normal
Choice 5- Excessive inclination may take too much medial bone, but if appropriately placed, would not risk injuring the rotator cuff insertion.

TSA:
Replacement of humeral head and glenoid resurfacing: cemented all-polyethylene glenoid resurfacing is standard of care

Total shoulder arthroplasty unique from THA and TKA in that:

  • Greater range of motion in the shoulder
  • Success depends on proper functioning of the soft tissues
  • Glenoid is less constrained: leads to greater sheer stresses and is more susceptible to mechanical loosening

Factors required for success of TSA
- Rotator cuff intact and functional: if rotator cuff is deficient and proximal migration of humerus is seen on x-rays (rotator cuff arthropathy) then glenoid resurfacing is contraindicated. If there is an irreparable rotator cuff deficiency then proceed with hemiarthroplasty or a reverse ball prosthesis. An isolated supraspinatus tear without retraction can proceed with TSA - incidence of full thickness rotator cuff tears in patients getting a TSA is 5% to 10% - if positive impingement signs on exam, order a pre-operative MRI.
- Glenoid bone stock and version
if glenoid is eroded down to coracoid process then glenoid resurfacing is contraindicated (Walch classification).

Outcomes
Pain relief more predictable than hemiarthroplasty, reliable range of motion, good survival at 10 years (93%), good longevity with cemented and press-fit humeral components, worse results for post-capsulorrhaphy arthropathy.

Indications

  • Pain (anterior to posterior), especially at night, and inability to perform activities of daily living
  • Glenoid chondral wear to bone: preferred over hemiarthroplasty for osteoarthritis and inflammatory arthritis
  • Posterior humeral head subluxation

Contraindications

  • Insufficient glenoid bone stock
  • Rotator cuff arthropathy
  • Deltoid dysfunction
  • Irreparable rotator cuff (hemiarthroplasty or reverse total shoulder are preferable) as risk of loosening of the glenoid prosthesis is high (“rocking horse” phenomenon)
  • Active infection
  • Brachial plexus palsy

Glenoid loosening - most common cause of TSA failure (30% of primary OA revisions)
Risk factors: insufficient glenoid bone stock (posterior glenoid wear associated with glenoid loosening), rotator cuff deficiency
2.9% reoperation rate for loosening (28% with revision)

Vascular injury
Arcuate artery, branch off the anterior humeral circumflex artery, can be damaged during biceps tendon elevation

Humeral stem loosening: more common in RA and osteonecrosis. Rule out infection.

Subscapularis repair failure

Malposition of components

Improper soft tissue balancing: failure due to undiagnosed presence of rotator cuff tears

Iatrogenic rotator cuff injury can occur if humeral neck osteotomy is inferior to level of rotator cuff insertion. Overstuffing glenohumeral joint leading to attritional supraspinatus and subscapularis tears

Stiffness

Infection: may have normal aspiration results
culture. Infection rate 1-2% after primary TSA.
Arthroscopic tissue culture more sensitive (100% sensitive and specific) than fluoroscopically guided aspiration (17% sensitivity, 100% specific)
Propionibacterium acnes (P. acnes),
now referred to as Cutibacterium acnes (c. acnes) most common cause of indolent infections and implant failures - gram positive, facultative, aerotolerant, anaerobic rod that ferments lactose to propionic acid
has high bacterial burden around the shoulder forms biofilm.
P. acnes PJI more common in males
Use anaerobic culture bottles, keep for 10-14days (mean time to detection 6 days)

Neurologic injury: axillary nerve is most commonly injured, musculocutaneous nerve can be injured by retractor placement under conjoint tendon

Periprosthetic fracture: acceptable fragment alignment ≤ 20° flexion/extension, ≤ 30° varus/valgus, ≤ 20° rotation malalignment

Wright and Cofield system:
Type A fractures are proximal to the stem tip and are treated with ORIF;
Type B fractures are at the level of the stem tip and are treated with ORIF;
Type C fractures are distal to the stem tip and can be initially treated nonoperatively.

Steinmann et al. reviewed the treatment of periprosthetic humeral shaft fractures.

For intraoperative fractures, the authors recommended placement of a long stem prosthesis that bypasses the fracture site by at least 2 cortical diameters.
For postoperative type A and B fractures, treatment depends on whether the stem is loose or well fixed. Loose prostheses necessitate revision long stem component with supplementary fixation, whereas well-fixed stems require hybrid plate fixation. Type C fractures can be treated non-operatively, but in the presence of nonunion may require plate fixation with or without allograft struts.

How well did you know this?
1
Not at all
2
3
4
5
Perfectly
14
Q

A 26-year-old accountant has recurrent shoulder instability. His first dislocation occurred after a fall while skiing. He has now sustained his third dislocation, which was reduced in the emergency department prior to being sent to your office. What is the most appropriate definitive treatment?

FIGURES: A : CT recon showing small (~10%) bony defect on anterior glenoid.

Immobilization in external rotation for 6 weeks
Arthroscopic bony Bankart repair
Arthroscopic Remplissage procedure
Glenoid augmentation using coracoid transfer
Glenoid augmentation using tricortical iliac crest graft

A

Arthroscopic bony Bankart repair

This patient has recurrent shoulder instability with a small bony defect of the anterior glenoid and no previous surgery. The most appropriate definitive management in this patient would be arthroscopic bony Bankart repair.
Figure A shows an en face sagittal 3D reconstruction of a glenoid with 10% surface area loss.

Older (>20 years old), recreational athletes with minor glenoid bone loss (<20% of the glenoid surface area) may be treated with soft tissue stabilization procedures using suture anchors. Goals of this procedure include tightening and repairing the torn ligament and labrum to the glenoid. Younger, contact sports athletes with large glenoid defect (>20%) may require bony augmentation type of procedures.

Defects larger than 25% of glenoid width should be managed with bony augmentation, with soft-tissue stabilization in smaller defects.

Following Bankart procedure risk factors for failure: age <=20, competitive participation in contact sports, shoulder hyperlaxity, Hill-Sachs on AP radiograph, glenoid bone loss of contour on AP radiograph.

The MOON Shoulder Group compared radiography, MRI and CT to determine the most reliable imaging modality for predicting bone loss. Three-dimensional CT, followed by regular CT were the most reliable and reproducible imaging modalities for predicting glenoid bone loss.

Incorrect Answers:
Answer 1: While closed reduction and immobilization are appropriate initial management, after failing conservative management, definitive management for recurrent shoulder dislocation is surgical.
Answer 3: Remplissage procedure is indicated in setting of large Hill-Sachs lesions.
Answers 4 & 5: Glenoid augmentation procedures using coracoid transfer (Bristow-Latarjet) and tricortical iliac crest graft would be indicated in setting of larger glenoid bone loss.

Arthroscopic Bankart Repair +/- capsular plication:
- Relative implications: 1st time dislocation with Bankart on MRI in athlete <25 years, high demand athletes, recurrent dislocation/subluxation (>1 dislocation) following non-operative management <20-25% bone loss, remplissage augmentation with arthroscopic Bankart may be considered if Hills-Sachs “off-track”.
Technique: at least 3 anchor points should be used, paramount that labrum is fully mobilised prior to repair.
Equal outcomes to open repair with advantage of less pain and greater motion preservation.
Failure rates increased in patients with global hyper laxity, glenoid bone loss or too few fixation points.

Laterjet (coracoid transfer) or Bristow Procedure:

  • For chronic bony deficiencies with >20-25% glenoid deficiency (inverted pear deformity to glenoid).
  • Transfer of coracoid bone with attached conjoined tendon and CA ligament
  • Laterjet has the triple effect = bony (increases glenoid track), sling (conjoined tendon on top of subscap), capsule reconstruction (CA ligament)
  • Deltopectoral approach, split subscap
Consider autograft (tri-cortical Iliac crest or distal clavicle) or allograft (iliac crest or distal tibia) for bony deficiencies with >20-25% glenoid deficiency (inverted pear deformity to glenoid) and revision of failed latarjet. 
Can be performed arthroscopic or open. 

Remplissage and Bankart Repair:
- For engaging large (>25-40%) Hill-Sach or ‘off-track’ Hill-Sach lesion with <20-25% glenoid bone loss
- Technique: posterior capsule and infraspinatus tendon is sutured into the Hill-Sach lesion, may be performed with concomitant Bankart repair. By decreasing size of Hill-Sach, converts off-track lesion into on-track lesion.
When compared to latarjet at 2 years, replissage and Bankart has lower recurrent instability rates (1.4% vs 3.2%) despite greater bipolar bone loss.

Bone graft recon for Hill-Sachs Defect
- For engaging large (>40%) Hill-Sach lesion
Technique: allograft reconstruction, arthroplasty, rotational osteotomy

Consider tendon transfer for chronic irreparable subscapularis tears (lat dorsi or sternal head of pec major)

How well did you know this?
1
Not at all
2
3
4
5
Perfectly
15
Q

A patient with shoulder pain and weakness has an MRI showing a cyst in the suprascapular notch. Which of the following muscles is most likely to show weakness?

Deltoid
Supraspinatus
Supraspinatus and infraspinatus
Infraspinatus
Teres minor
A

Supraspinatus and infraspinatus

The suprascapular notch is proximal to the point where the suprascapular nerve innervates both the supraspinatus and the infraspinatus, therefore compression would cause weakness of both.

Compression at the spinoglenoid notch will affect only the infraspinatus as the suprascapular nerve has already innervated the supraspinatus by this point. The teres minor and deltoid are both innervated by the axillary nerve. The axillary nerve passes through the quadrangular space and compression here could result in denervation of the posterior deltoid. Spinoglenoid notch cysts are classically seen in volleyball players and associated with SLAP tears.

Martin et al review the outcomes of 24 overhead athletes who underwent arthroscopic debridement of labral tears. Results were good to excellent in 21 of the 24 patients when there was no gross instability or Bankart lesion present.

Meister et al review the evaluation and treatment of the throwing athlete. Rotator cuff weakness, labral tears and paralabral cysts are discussed. Treatment is based on the pathoanatomy of the throwing shoulder, and most athletes will achieve successful rehabilitation with nonoperative care.

Suprascapular neuropathy is compression of the suprascapular nerve that most commonly occurs at the suprascapular notch or spinoglenoid notch by a mass (i.e cyst).
Diagnosis can be suspected clincally with weakness and atrophy of the infraspinatous or supraspinatous and confirmed with MRI studies showing cysts in the suprascapular notch or spinoglenoid notch.
Treatment of suprascapular nerve compression at the suprascapular notch requires decompression of a cyst when present. Treatment of a spinoglenoid cyst requires either decompression or repair of an associated labral lesion (if present).

Suprascapular notch entrapment:
Proximal compression of supra scapular nerve in the supra scapular notch leads to weakness of both supraspinatus and infraspinatus. Suprascapular nerve (C5,6) - emerges off superior trunk of brachial plexus, travels across posterior triangle of neck to scapula innervation of supraspinatus and infraspinatus. Compression can be due to a ganglion cyst (often associated with labral tears), transverse scapular ligament entrapment, fracture callus.

Spinoglenoid notch entrapment:
Distal compression of supra scapular nerve therefore weakness of infraspinatus only. Compression can be due to posterior labral tears -> cyst, spinoglenoid ligament, spinoglenoid notch ganglion, traction injury (volleyball players), transglenoid fixation

Suprascapular ligament arises from medial base of coracoid and overlies suprascapular notch - suprascapular artery runs above and suprascapular nerve runs below.

Spinoglenoid ligament arises near spinoglenoid notch and overlies distal suprascapular nerve

How well did you know this?
1
Not at all
2
3
4
5
Perfectly
16
Q

A 49-year-old man sustains a dislocation of his left elbow that is successfully reduced and splinted. He misses his scheduled follow-up appointments and returns 6 weeks later. He is immediately enrolled in a course of vigorous physical therapy. At a repeat visit at 6 months, examination reveals that he lacks 40 degrees of elbow extension, and has flexion to 80 degrees. He is taken to the operating room for surgical release. To restore elbow flexion, in addition to releasing the articular capsule, which ligament should be released?

Anterior Band MCL
Posterior band MCL
Transverse band MCL
Radial Collateral ligament
LUCL
A

Posterior band MCL

In addition to capsular release, the posterior band of the medial collateral ligament (MCL) should be released.

The posterior band of the MCL is attached dorsal to the axis of rotation and has greater variation in length. It increases in length by 9 mm between 60° and 120° of flexion. Posterior band contracture leads to loss of elbow flexion. In contrast, the anterior band of the MCL (AMCL) maintains a constant length (isometric) throughout the entire arc of movement. Anterior capsule contracture leads to loss of extension.

Wada et al. treated 14 elbows with post traumatic contracture. Through a medial incision, the ulnar nerve was freed and the posterior band and posteromedial joint capsule were excised. Mean flexion increased from 89° preop to 127° postop. Anterior capsulectomy was performed for limited extension.

Morrey et al. studied structures providing stability about the elbow. They found that the anterior capsule stabilizes the elbow to varus-valgus stress in extension, not in flexion. The anterior band of the MCL is a primary stabilizer, especially in flexion.

Incorrect Answers:
Answer 1: This is the anterior band of the medial collateral ligament and should not be released. Release of the AMCL leads to valgus instability.
Answer 3: This is the transverse band of the medial collateral ligament. Release of this ligament will not improve elbow ROM.
Answer 4: This is the radial collateral ligament and should not be released.
Answer 5: This is the lateral ulnar collateral ligament (LUCL) and should not be released. Release of the LUCL leads to posterolateral rotatory instability (PLRI).

The medial UCL is subjected to near-failure tensile stresses during the late cocking/early acceleration phase of throwing.

The medial ulnar collateral ligament, or medial collateral ligament of the elbow, is composed of three bundles:

  • an anterior bundle
  • a posterior bundle
  • a variable transverse oblique bundle.

The anterior bundle of the ulnar collateral ligament is the primary restraint to valgus force of the elbow from 30 to 120 degrees of flexion.

Elbow Stiffness and Contractures of the elbow result in loss of motion and difficulty performing activities of daily living and may occur as a result of trauma, osteoarthritis, elbow surgery, or a congenital condition.
Diagnosis is made clinically with assessment of active and passive elbow range of motion with a comparison to the contralateral side.
Treatment is a trial of aggressive physical therapy to achieve functional range of motion. Operative management is indicated in the event of bony block to motion, congenital disease and lack of improvement with physical therapy.

Patients are able to perform ADLs if elbow ROM 30deg (ext) to 130 (flexion) is achieved. Most activities require a 100 deg arc of motion at the elbow to be functional, 30deg loss of extension is well tolerated, a flexion loss causes more dysfunction than extension loss.

Capsular release +/- release of posterior band of MCL
- For extrinsic capsular contractures with normal joint surface congruency (most predictable beneficial results), less predictable in patients with arthritis (joint surface is incongruous).
Compliance with post-op rehab is critical.
Contraindicated in charcot elbow, neurological elbow, poor skin condition (may need rotational flap)
Arthroscopic = technical demanding, places radial nerve at risk (portal placement)
Open:
- Lateral column approach (Morrey) - elevate ECRL & BR anteriorly, triceps posteriorly, mobilise brachial off anterior capsule, debride/release anteriorly and posteriorly, including coronoid tip/fossa, olecranon tip/fossa, anterior and posterior capsule and radiocapitellar joint.
- Medial ‘over the top’ column approach (Hotchkiss) - best for extrinsic contractures, MCL calcifications and/or baseline ulnar nerve symptoms - perform with decompression or transposition of ulnar nerve. Release posterior band MCL to increase flexion, then working anterior to flexor-pronator mass, decried/release anteriorly including coronoid tip/fossa and anterior capsule.

Timing of release: consider contracture release at 4-6 months post injury/surgery if ROM plateau’d and appropriate splinting/therapy has been performed
Post-op complications: Heterotopic ossification, transient ulnar neuropraxia, ulnar nerve damage, recurrent contracture.

Osteophyte excision : intrinsic contracture with arthritis confined to olecranon fossa - perform in conjunction with capsular release of bony block to terminal ROM - bone typically should be removed from coronoid, coronoid fossil olecranon and olecranon fossa.

Distraction interposition arthroplasty
- For intrinsic contractures with diffuse arthritis in high demand, younger patients.

TEA
- Intrinsic contractures with diffuse arthritis in low demand elderly patients - high failure rate in young, active patients, imposes a permanent 2kg lifting restriction.

17
Q

What structure provides dynamic glenohumeral stability by compressing the humeral head against the glenoid?

Superior glenohumeral ligament
Middle glenohumeral ligament
Teres major muscle
Deltoid muscle
Rotator cuff muscles
A

Rotator cuff muscles

The rotator cuff is the main DYNAMIC stabilizer of the glenohumeral joint. It functions most at midrange motion, not at the extremes of range of motion. The superior glenohumeral ligament is a STATIC stabilizer and resists inferior translation at 0° degrees of abduction. The middle glenohumeral ligament is a STATIC stabilizer and resists anterior translation in the midrange of abduction (~45°) in ER. The teres major adducts and medially rotates arm and is not a significant stabilizer of the glenohumeral joint. The deltoid muscle primarily abducts the arm and is not the major stabilizer of the glenohumeral joint.

Glenohumeral stability:

  • Static restraints: glenohumeral ligaments (SGHL, MGHL, IGHL - posterior, anterior & superior bands), glenoid labrum, articular congruity and version, negative intraarticular pressure
    (if release head will sublux inferiorly)
  • Dynamic restraints: rotator cuff, rotator interval, biceps long head, periscapular muscles
  • – Rotator cuff muscles: the primary biomechanical role of the rotator cuff is stabilizing the glenohumeral joint by compressing the humeral head against the glenoid

SGHL: from anterosuperior labrum to humerus, provides restraint to inferior translation at 0 deg abduction, prevents anteroinferior translation of long head of biceps (biceps pulley).

MGHL: resists anterior and posterior translation in the mid-range of abduction (~45deg) in ER

IGHL:

  • Posterior band: restraint posterior subluxation at 90deg flexion and IR - tightness leads to internal impingement and increased shear forces on superior labrum (linked to SLAP lesions)
  • Anterior band: primary restraint to anterior/inferior translation 90deg abduction and max ER (late cocking phase of throwing), anchors into anterior labrum and forms a weak link that predisposes to Bankart lesions.
  • Superior Band: most important static stabiliser about the joint, 100% increased strain on superior band of IGHL in presence of SLAP lesion.
18
Q

An 18-year-old high school volleyball player is being treated for multidirectional instability of the right shoulder with a physical therapy program. She has intermittent pain and instability and episodic numbness and weakness in the ipsilateral hand. All of the following are characteristic features of a generalized connective tissue disorder EXCEPT:

Elbow hyperextension of the left arm
Left 5th finger passive extension beyond 90°
Genu recurvatum of the bilateral knees
Excessive supination of the left forearm
Abducted thumb to reach the ipsilateral forearm (thumb-to-forearm test) of the right hand

A

Excessive supination of the left forearm

Excessive supination of the left arm is not listed as part of the Beighton 9-point scoring system for hypermobility. All of the other options are part of this scoring system, and a score of >6 is associated with connective tissue disorders such as Marfan’s and Ehlers-Danlos Syndrome.

Multidirectional shoulder instability (MDI) is a condition characterized by generalized instability of the shoulder in at least 2 planes of motion (anterior, posterior, or inferior) due to capsular redundancy.
Diagnosis is made clinically with presence of increased anterior and posterior humeral translation, a sulcus sign, and overall increased external rotation.
Sulcus sign assesses rotator interval - laxity of rotator interval presents as increased external rotation with the aim fully adducted and at 90 deg abduction.
Apprehension/relocation test
Anterior and posterior load and shift test
Neer and Hawkins test - impingement of rotator cuff tendonitis in <20 year old signals possible MDI.
Assess for signs of generalised hypermobility - ligamentous laxity i.e. Beighton’s criteria >4/9:
- Able to touch palms to floor while bending at waist (1 point)
- Genu recurvatum (2 points)
- Elbow hyperextension (2 points)
- MCP hyperextension (2 points)
thumb abduction to the ipsilateral forearm (2 points)
Treatment is a trial of prolonged physical therapy focusing on dynamic stabilization and periscapular muscle training. Arthroscopic stabilization with capsular shift is indicated for patients with persistent instability who fail an extensive course of physical therapy.

The Level 5 review article by Schenk and Brems notes that generalized ligamentous laxity has been reported in 45% to 75% of patients who have undergone surgery for multidirectional (MDI) shoulder instability. Patients with MDI have pathologic laxity of the glenohumeral joint in more than one direction with at least one of those being inferior. The onset of symptoms is frequently atraumatic, and the chief complaint is often pain more than instability. Patients can experience concomitant recurrent, transient episodes of numbness, tingling, and weakness in the affected extremity. Most patients can be successfully treated nonoperatively with a specific exercise program. If a 6-month trial of nonoperative management fails, then surgical intervention with an inferior capsular shift can be performed.

Glenohumeral stability:

  • Static restraints: glenohumeral ligaments (SGHL, MGHL, IGHL - posterior, anterior & superior bands), glenoid labrum, articular congruity and version, negative intraarticular pressure
    (if release head will sublux inferiorly)
  • Dynamic restraints: rotator cuff, rotator interval, biceps long head, periscapular muscles
  • – Rotator cuff muscles: the primary biomechanical role of the rotator cuff is stabilizing the glenohumeral joint by compressing the humeral head against the glenoid

SGHL: from anterosuperior labrum to humerus, provides restraint to inferior translation at 0 deg abduction, prevents anteroinferior translation of long head of biceps (biceps pulley).

MGHL: resists anterior and posterior translation in the mid-range of abduction (~45deg) in ER

IGHL:

  • Posterior band: restraint posterior subluxation at 90deg flexion and IR - tightness leads to internal impingement and increased shear forces on superior labrum (linked to SLAP lesions)
  • Anterior band: primary restraint to anterior/inferior translation 90deg abduction and max ER (late cocking phase of throwing), anchors into anterior labrum and forms a weak link that predisposes to Bankart lesions.
  • Superior Band: most important static stabiliser about the joint, 100% increased strain on superior band of IGHL in presence of SLAP lesion.
19
Q

A 24-year-old bodybuilder reports shoulder pain after an injury while bench pressing. Physical exam reveals ecchymosis and swelling in his right upper arm. He has weakness in internal rotation but has good strength in external rotation and abduction; his apprehension test is negative. When he puts his hands on his hips, his upper chest is asymmetrical. When is surgery indicated for this injury?

Surgery is not indicated; conservative management including ice, rest and NSAIDs are recommended
After a period of immobilization, followed by physical therapy, has failed
When the pectoralis major has avulsed from its humeral insertion
Asymmetry of the upper chest wall without functional deficits
If swelling and ecchymosis are primarily located on the chest wall rather than the upper arm

A

When the pectoralis major has avulsed from its humeral insertion

The injury described in this question is consistent with is a humeral avulsion of the pectoralis major muscle, which should be treated with primary surgical repair.

Surgical repair is indicated with complete pecto-humeral tears. Tears are often found on history and examination, however the location of tear often is determined by radiographic adjuncts such as MRI. The standard of practice includes suture anchors or bone tunnels into the pectoralis insertion on the humerus.

Schepsis et al. looked at a retrospective case series of 17 patients with acute and chronic distal pectoralis major muscle rupture. They showed that outcome measures of strength after surgical repair of acute and chronic tears were not significant. However, patients treated operatively fared significantly better than patients treated non-operatively.

Pochini et al. looked at a prospective cohort of 60 patients treated conservatively vs operatively for pectoralis major muscle rupture. They found that the non-operatively group had more complications and decrease in strength of 41.7% relative to 4.3% for the surgical group.

Figure A shows bruising in the right axilla consistent with physical exam findings of a pectorals major tear.
T2 weighted MRI will show a retracted pectoralis major tendon, periosteum pulled off the humeral cortex adjacent to the biceps tendon and hemorrhage within the muscle.

Incorrect Answers:
Answer A and B: Operative outcomes show significantly better results than non-operative outcomes.
Answer D: Asymmetry of the chest wall can result from a spectrum of pectorals major tears, including bony tendon avulsions, mid-tendon and musculotendinous tears. Asymmetry of the chest wall alone, does not warrant surgical intervention. Tear location, function and patient factors must also be considered.
Answer E: Swelling and ecchymosis over the chest wall is likely indicative of a complete or incomplete mid-substance pectoralis major tear. These injuries typically do not benefit from surgery.

A rare acute injury caused by avulsion of the pectoralis major tendon and usually seen in weightlifters.
Diagnosis is generally made clinically and is confirmed with MRI studies.
Treatment is usually surgical repair when presenting acutely.

Mechanism
Excessive tension on a maximally eccentrically contracted muscle - occurs during the downward portion of a bench press, with the arm in the final 30 degrees of humeral extension while pushing against heavy resistance.
Tendon fails in a predictable sequence
inferior fibers of sternocostal head fail first then superior fibers of the sternocostal head finally the clavicular head.

Pec Maj: Lateral pectoral nerve (C5-7) supplies lower muscle belly, whilst medial pectoral nerve (C8-T1) supplies upper portion.
2 heads at origin:
- Clavicular head: medial clavicle and proximal sternum
- Sternocostal head: distal sternum, costal cartilage, ribs 1-6, external oblique aponeurosis - (larger portion >80% of muscle volume).
Inserts on the humeral shaft just lateral to the bicipital groove
Action: shoulder adduction and IR, clavicular head contributes some forward flexion.

Modified Tietjen (anatomic) classification exists

I Muscle contusion or sprain
II Partial tear
III Complete tear (further subclassified by location) A - muscle origin, B - muscle belly, C- musculotendinous junction, D intratendinous junction, E - tendon avulsion off humerus (no bone), F - bony avulsion off humerus.

Open primary repair is the gold standard for acute tears in young active patients and high level athletes, especially for tendon avulsion and myotendinous junction tears.
Standard deltopectoral approach - can use transosseous suture repair with cortical trough, cortical button fixation, suture anchor repair.

Reconstruction for chronic tears that cannot be adequately mobilised for primary repair.
Deltopectoral approach, release adhesions - care to avoid injury to medial and lateral pectoral nerves. Supplemental fascial release may be necessary to mobilise the muscle belly in chronic situations.
Graft options: Achilles allograft, gracilis weave (allograft vs autograft).

20
Q

Which of the following is true of the scapula during an overhead throwing motion?

It maximally retracts on ball release
It protracts during late cocking to prevent impingment on the rotator cuff
It must rotate in the cocking and acceleration phases to prevent impingement on the rotator cuff
It must remain fixed during the throwing motion to impart maximal energy
It has no effect on concavity-compression

A

It must rotate in the cocking and acceleration phases to prevent impingement on the rotator cuff

The scapula has an integral role in the overhead throwing motion. It must rotate during cocking and acceleration to clear the acromion to prevent impingement on the rotator cuff.

5 important roles of the scapula during throwing:

  1. provide a stable glenohumeral articulation;
  2. retraction and protraction;
  3. elevation of the acromion;
  4. serve as a base for muscular attachment;
  5. to be a link in the kinetic chain.

Burkart et al. created the acronym “SICK” scapula (Scapular malposition, Inferior medial border prominence, Coracoid pain and malposition, and dysKinesis of scapular movement) to describe common signs and symptoms in the disabled throwing shoulder. This is felt to be an overuse muscular fatigue syndrome and is a recognized cause of shoulder pain in the throwing athlete.

Incorrect Responses:

  1. & 2. The scapula maximally retracts during late cocking and protracts during acceleration and ball release (through activity of the serratus anterior muscle, which is innervated by the long thoracic nerve).
  2. The scapula is dynamic and normal motion is needed to generate maximal force transfer.
  3. The scapula keeps the humeral head centered thereby increasing concavity-compression and therefore stability.

5 main phases of throwing:
- Wind up - minimal force on shoulder during first stage, RC muscles are inactive during this phase.

  • Cocking:
    Early : peak muscle activation is deltoid.
    Late : high torque phase with maximal shoulder ER, high elbow valgus stress, peak muscle activation: supraspinatus, infraspinatus, teres minor. FDS and FCU muscles provide local dynamic stability of the elbow during late cocking (protective against injury to UCL)
  • Acceleration: rotates ball to release point - early muscle activation = triceps, late muscle activation = pec maj, lat dorsi, serratus anterior,
  • Deceleration: centre of gravity moves over plantar foot. Eccentric contraction of all muscles is required to slow down arm motion
    Highest torque phase
    Recognised as the most harmful phase of throwing - associated with SLAP lesion, biceps tendon injury, brachialis injury and tires minor injury.
  • Follow through: where body rebalances and stops forward motion. Muscle activity returns to resting level.

Scapula must work in concert with humerus to maintain glenohumeral stability.
The entire throwing motion takes approx 2 seconds - the wind up and acceleration phases last ~75% of the time (1.5seconds)

21
Q

A minor league baseball player presents with deteriorating pitch velocity and control in addition to worsening elbow soreness. His T2 coronal MR sequence shows a torn MUCL and based on this he decides to proceed with surgery. Which of the following medial ulnar collateral reconstruction techniques would give this athlete the best chance to return to sport?

Modified Jobe technique and ulnar nerve submuscular transposition
Classic Jobe technique and ulnar nerve in situ release
Classic Jobe technique and ulnar nerve transposition
Docking technique and ulnar nerve in situ release
Modified Jobe technique and medial epicondylectomy

A

Docking technique and ulnar nerve in situ release

The docking technique for medial ulnar collateral ligament (MUCL) reconstruction with a muscle splitting approach has been shown in multiple studies to have the highest rate of return to sport. In situ ulnar nerve release is most commonly performed in the absence of pre-operative neuropathy.

The medial ulnar collateral ligament (MUCL) is the primary restraint to valgus across the elbow, with the anterior bundle contributing the most to stability. Multiple different reconstruction techniques have been developed, yet all are aimed at reconstructing the anterior bundle of the MUCL using a tendon graft.

Watson et al. reviewed MUCL reconstruction techniques among overhead throwing athletes. They found the highest return to play rate with the docking technique (90%) compared to the classic Jobe (67%) and modified Jobe technique (77%). However, this difference may be confounded by the adoption of muscle splitting techniques, fewer obligatory ulnar nerve transpositions, and advances in rehabilitation.

Vitale and Ahmad also reviewed MUCL reconstruction techniques. They found avoiding obligatory ulnar nerve transposition had a significantly lower rate of post-operative neurapraxia (3% vs 8%) and better outcomes (90% excellent vs 76% excellent results). This study highlights the positive effects of research and refinement of MUCL reconstruction techniques.

Arner et al. compared cohorts who underwent either the docking technique or the modified Jobe technique at a single institution. They reported similar KJOC scores, Conway scores, and rates complications and return to sport between both cohorts. These findings may show there to be fewer differences between these techniques when interchangeably performed by a single, experienced surgeon.

Incorrect Answers:
Answer 1: Routine ulnar nerve transposition is not advised without pre-operative neuropathy.
Answers 2 and 3: The classic Jobe technique has fallen out of favor and is done sparingly.
Answer 5: Medial epicondylectomy should be avoided in throwing athletes.
Ulnar collateral ligament (UCL) reconstruction using a flexor-pronator muscle-splitting approach and a docking graft fixation technique are associated with the lowest complication rate and best patient outcomes.

Vitale et al. demonstrated that the flexor-pronator muscle-splitting approach was associated with better outcomes than detachment of the flexor-pronator mass, had a lower rate of postoperative ulnar neuropathy, and a lower overall complication rate. They also found fixation of the graft utilizing the docking technique was associated with better outcomes than the figure-of-8 technique. Abandoning the obligatory ulnar nerve transposition was associated with improved patient outcomes (89% vs. 75%) and a lower rate of postoperative ulnar neuropathy (4% vs. 9%).

Rettig et al performed a case series review of 31 overhead throwing athletes with ulnar collateral ligament injuries managed nonoperatively with 3 months rest followed by rehabilitation exercises. They concluded that 42% of athletes were able to return to their previous level of competition at an average of 6 months from diagnosis (earlier than reconstruction). The authors were unable to identify any patient-specific factors (duration of symptoms, age, acuity of onset) that would predict the success of nonoperative treatment.

In the docking graft fixation technique the graft is placed in a triangular configuration through a single humeral tunnel. The suture limbs are then brought out through two separate bone holes and tied over a bony bridge on the superior aspect of the medial epicondyle.

The figure-of-8 (Jobe) graft fixation technique is performed by passing the tendon graft through two bone tunnels in the medial epicondyle of the humerus and through one tunnel in the ulnar sublime tubercle. The graft is then sutured to itself in a figure-of-8 configuration.

Incorrect Answers:
Answer 1: The figure-of-8 technique is not associated with better patient outcomes when compared to the docking technique.
Answer 3: Obligatory ulnar nerve transposition during UCL reconstruction is associated with a higher rate postoperative ulnar neuropathy and worse patient outcomes, and therefore should be avoided.
Answers 4 and 5: Detachment of the flexor-pronator mass is not associated with better patient outcomes when compared to the muscle-splitting approach.

The medial ulnar collateral ligament, or medial collateral ligament of the elbow, is composed of three bundles:

  • an anterior bundle
  • a posterior bundle
  • a variable transverse oblique bundle.

The anterior bundle of the ulnar collateral ligament is the primary restraint to valgus force of the elbow from 30 to 120 degrees of flexion.

Biomechanical testing has shown that valgus forces as high as 64 N.m at the elbow during late cocking and early acceleration phases of throwing with compressive forces of 500 N at the lateral radiocapitellar articulation as the elbow moves from 110 to 20 degrees of flexion and velocities as high as 3000 deg/sec.

Mechanisms of injury MUCL:
- Acute trauma: often associated with elbow dislocations
- Overuse injuries:
+ Microtrauma from repetitive valgus stress leads to rupture of anterior band of MUCL
+ Baseball pitchers place significant valgus stress on the elbow in the late cocking and early acceleration phase of throwing
+ elbow valgus load increases with poor throwing mechanics and decreases with trunk-scapular kinesis, forearm pronation, dynamic flexor-pronator stabilization

  • Iatrogenic : excessive olecranon osteophyte resection places MCL at risk
22
Q

In valgus extension overload of the elbow, which is the typical location of osteophytes formation?

Lateral condyle
lateral margin radial head
posterolateral olecranon
posteromedial olecranon
medial condyle
A

Valgus extension overload syndrome of the elbow occurs in throwing athletes. The mechanism is thought to be valgus stress on the elbow during acceleration, especially with insufficiency of the medial ulnar collateral ligament, as well as extension impaction forces during decceleration. Over time, the continuous impaction of the posterior-medial olecranon in the olecranon fossa can lead to chondromalacia and osteophyte formation.

Wilson et al. described this ostoephyte formation in pitchers which were successfully treated with osteophyte removal. This can be done either arthroscopically or with a limited open arthrotomy.

Valgus Extension Overload, also known as Pitcher’s elbow, is a condition characterized by posteromedial elbow pain related to repetitive microtrauma in throwing athletes.
Diagnosis is made clinically with posteromedial elbow pain that worsens with elbow extension, and confirmed with radiographs showing osteophyte formation in the posteromedial olecranon fossa.
Treatment is usually nonoperative with rest, activity modifications, and injections. Operative resection of osteophytes and loose bodies are indicated in pitchers with persistent symptoms.

The repetitive stress of pitching leads to excessive shear forces on medial aspect of olecranon tip and olecranon fossa, lateral radio-capitellar compression, posterior extension overload and medial tension at MCL.

Leads to cartilage injury from repetitive impaction of olecranon into olecranon fossa, osteochondral lesions of the capitellum, osteophyte formation (posteromedial humerus and olecranon), loose bodies from fragmentation, MCL can be attenuated with repetitive strain.

Concurrent cubital tunnel syndrome occurs in ~25% of cases.

23
Q

Besides the biceps tendon, which of the following structures also pass through the rotator interval?

The coracohumeral ligament only
The coracohumeral and superior glenohumeral ligaments
The coracohumeral, superior and middle glenohumeral ligaments
The superior and middle glenohumeral ligaments
The superior glenohumeral ligament only

A

The coracohumeral and superior glenohumeral ligaments

The rotator cuff is perforated anterosuperiorly by the coracoid process, which separates the anterior border of the supraspinatus tendon from the superior border of the subscapularis tendon, creating the triangular rotator interval, which is bridged by capsule. The base of the interval is the coracoid process, from which capsular tissue (the coracohumeral ligament) originates. The transverse humeral ligament at the biceps intertubercular sulcus forms the apex of the rotator interval. The coracohumeral and superior glenohumeral ligaments are considered to be structural contents of the rotator interval capsule, but each have separate origins and insertions. These ligaments are considered to be the most constant structures of the fibrous joint capsule.

Arai et al performed cadaver dissections to describe the anatomy as it relates to reconstructing the biceps sling as it exits the interval in cases of biceps subluxation. They note that an intact superior border of subscapularis is needed as well as tension in the SGHL.

Yang et al reported a descriptive anatomy study on the CHL. All were located in the rotator interval, originated from the lateral aspect of the base of the coracoid process, and had histology more consistent with capsule than ligament.

Boundaries of the rotator interval include the coracoid process (COR) at its base, superiorly by anterior margin of supraspinatus tendon (SST) and inferiorly by superior margin of subscapularis tendon (SSC). Contents of rotator interval include long head of biceps tendon (BT), coracohumeral ligament (CHL), superior glenohumeral ligament (SGHL), and rotator interval capsule.
Rotator interval capsule (RIC) is the anterosuperior aspect of glenohumeral joint capsule, which merges with CHL and SGHL insertions medial and lateral to bicipital groove.

24
Q

A 24-year-old male sustains the right elbow open dislocation. He promptly undergoes operative irrigation and debridement, reduction, vascular bypass of the brachial artery, and hinged elbow fixator placement for 6 weeks. Three years later he complains of clicking and locking with elbow extension and difficulty performing arm triceps dips while attempting exercise. He is unable to perform a pushup. Which of the following reconstruction procedures is MOST appropriate?

Elbow arthroscopy with extensor tendon insertion debridement
Lateral ulnar collateral ligament reconstruction with palmaris tendon graft
Radial collateral ligament reconstruction with palmaris tendon graft
Medial ulnar collateral ligament reconstruction with palmaris tendon graft woven in a figure-eight fashion (Tommy John procedure)
Medial collateral ligament reconstruction with palmaris tendon graft woven using Docking procedure

A

Lateral ulnar collateral ligament reconstruction with palmaris tendon graft

This clinical pesentation is consistent with posterolateral rotatory instability of the elbow due to a previous open elbow dislocation. The most appropriate treatment is lateral ulnar collateral ligament reconstruction with palmaris tendon graft.

Posterolateral rotatory instability is caused by insufficiency to an important lateral staiblizer, the lateral ulnar collateral ligament (LUCL). Characteristic symptoms include clicking and locking with extension and difficulty getting out of a chair or performing tricep arm dips. The O’Driscoll lateral pivot shift test is peformed by having the patient lay supine with the extremity over the patient’s head. The shoulder is fully externally rotated and the examiner grasps the patient’s forearm and places it in full supination. Starting with supination and extension, the elbow is flexed with a valgus force and axial load being applied simultaneously. The most rotatory displacement and posterior displacement of the radial head is appreciated when the elbow is at approximately 40 degrees of flexion.

Sanchez-Sotelo et al present a retrospective review of 45 patients undergoing direct repair or reconstruction of the LUCL. They found that patients with subjective instability had better Mayo elbow performance scores, and reconstruction using a tendon graft seems to provide better results than ligament repair.

The medial UCL is subjected to near-failure tensile stresses during the late cocking/early acceleration phase of throwing.

The medial ulnar collateral ligament, or medial collateral ligament of the elbow, is composed of three bundles:

  • an anterior bundle
  • a posterior bundle
  • a variable transverse oblique bundle.

The anterior bundle of the ulnar collateral ligament is the primary restraint to valgus force of the elbow from 30 to 120 degrees of flexion.

Elbow Stiffness and Contractures of the elbow result in loss of motion and difficulty performing activities of daily living and may occur as a result of trauma, osteoarthritis, elbow surgery, or a congenital condition.
Diagnosis is made clinically with assessment of active and passive elbow range of motion with a comparison to the contralateral side.
Treatment is a trial of aggressive physical therapy to achieve functional range of motion. Operative management is indicated in the event of bony block to motion, congenital disease and lack of improvement with physical therapy.

Patients are able to perform ADLs if elbow ROM 30deg (ext) to 130 (flexion) is achieved. Most activities require a 100 deg arc of motion at the elbow to be functional, 30deg loss of extension is well tolerated, a flexion loss causes more dysfunction than extension loss.

Capsular release +/- release of posterior band of MCL
- For extrinsic capsular contractures with normal joint surface congruency (most predictable beneficial results), less predictable in patients with arthritis (joint surface is incongruous).
Compliance with post-op rehab is critical.
Contraindicated in charcot elbow, neurological elbow, poor skin condition (may need rotational flap)
Arthroscopic = technical demanding, places radial nerve at risk (portal placement)
Open:
- Lateral column approach (Morrey) - elevate ECRL & BR anteriorly, triceps posteriorly, mobilise brachial off anterior capsule, debride/release anteriorly and posteriorly, including coronoid tip/fossa, olecranon tip/fossa, anterior and posterior capsule and radiocapitellar joint.
- Medial ‘over the top’ column approach (Hotchkiss) - best for extrinsic contractures, MCL calcifications and/or baseline ulnar nerve symptoms - perform with decompression or transposition of ulnar nerve. Release posterior band MCL to increase flexion, then working anterior to flexor-pronator mass, decried/release anteriorly including coronoid tip/fossa and anterior capsule.

Timing of release: consider contracture release at 4-6 months post injury/surgery if ROM plateau’d and appropriate splinting/therapy has been performed
Post-op complications: Heterotopic ossification, transient ulnar neuropraxia, ulnar nerve damage, recurrent contracture.

Osteophyte excision : intrinsic contracture with arthritis confined to olecranon fossa - perform in conjunction with capsular release of bony block to terminal ROM - bone typically should be removed from coronoid, coronoid fossil olecranon and olecranon fossa.

Distraction interposition arthroplasty
- For intrinsic contractures with diffuse arthritis in high demand, younger patients.

TEA
- Intrinsic contractures with diffuse arthritis in low demand elderly patients - high failure rate in young, active patients, imposes a permanent 2kg lifting restriction.

25
Q

A 47-year-old, healthy, active patient presents with a sub-acute, full-thickness supraspinatus tear. His physical examination reveals significant weakness and pain with abduction. There was no glenohumeral instability. Radiographs demonstrate a type 1 acromion. An MRI scan shows a crescent shaped tear with 2-cm of tendinous retraction and no tendinous fatty changes. A subacromial corticosteroid injection 6 weeks ago provided him with 24 hours of pain relief but no improvement in strength. What would be the most appropriate treatment option?

Repeat subacromial corticosteriod injection
Biological augmentation of rotator cuff with porcine small intestine xenograft
Rotator cuff repair
Rotator cuff repair plus acromioplasty
Rotator cuff repair, remplissage procedure, bicep tenodesis and distal clavicle excision

A

Rotator cuff repair

This patient has an isolated supraspinatus rotator cuff tear with symptomatic weakness. The most appropriate treatment would be isolated rotator cuff repair.

The primary purpose of rotator cuff repair is to restore muscle function. Secondary outcomes include reduction of pain and prevention of irreversible cuff changes, specifically muscular atrophy. Non-operative treatment (exercise, therapy and pain medications) are recommended for partial thickness tears. The indication of surgical repair includes, isolated supraspinatus weakness +/- pain that correlates with MRI imaging of a respective full thickness tear. Routine acrominoplasty is not recommended in conjunction with rotator cuff repair, especially with no previous symptoms of impingement.

Pedowitz et al. developed clinical practice guidelines for the treatment of rotator cuff pathology. The strongest supporting evidence in current literature was given a grade of ‘moderate’ with four treatment recommendations. These were,

  1. Exercise and non-steroidal anti-inflammatory drugs can be used to manage partial thickness tears,
  2. Routine acromioplasty is not required the time of cuff repair,
  3. Non-cross-linked, porcine small intestine submucosal xenograft patches should not be used to manage cuff tears, and
  4. Surgeons can advise patients that workers’ compensation status correlates with a less favorable outcome after rotator cuff surgery.

Incorrect Answers:
Answer 1- Repeat subacromial corticosteriod injection could be considered, but this would not address this patients weakness.
Answer 2- There was moderate evidence against the use xenografts for full thickness rotator cuff repairs.
Answer 4- There was moderate evidence AGAINST routine acromioplasty in junction with rotator cuff repair, especially with no previous symptoms of impingement.
Answer 5- The remplissage procedure, bicep tenodesis and distal clavicle excision would not be warranted in this patient, as there is no supporting features in the clinical exam or imaging to support any additional pathology in the shoulder.

Double row suture techniques (mattress sutures in medial row and simple sutures in lateral row) have been shown to create a more anatomic repair of the footprint
lower retear rate compared with single row
no difference in functional score, pain score, time to healing (compared to single row)

Tendon transfer is indicated in massive cuff tears - e..g. pec maj or lat dorsi transfers (best for irrepairable posterosuperior tears with intact subscap).

The primary function of the rotator cuff is to provide dynamic stability by balancing the force couples about the glenohumeral joint in both the coronal and transverse plane.

Anatomic features associated with rotator cuff:

  • Rotator interval: includes the capsule, long head of the biceps tendon, SGHL, and the coracohumeral ligament that bridge the gap between the supraspinatus and the subscapularis.
  • Rotator crescent: thin, crescent-shaped sheet of rotator cuff comprising the distal portions of the supraspinatus and infraspinatus insertions.
  • Rotator cable: thick bundle of fibers found at the avascular zone of the coracohumeral ligament running perpendicular to the supraspinatous fibers and spanning the insertions of the supra- and infraspinatus tendons.
26
Q

A 10-year-old little league pitcher has the triad of medial elbow pain in his throwing arm, decreased throwing effectiveness, and decreased throwing distance. What is the pathogenesis of the condition that is most likely to be occuring in this patient?

Acute fragmentation of the entire capitellar ossific nucleus
Rupture of the anterior band of the anterior bundle of the ulnar collateral ligament
Repetitive contraction of the flexor-pronator mass stresses the chondro-osseous origin, leading to apophysitis
Microtraumatic vascular insufficiency of the capitellum from chronic compressive and rotatory forces
Repetitive triceps contraction during extension

A

Repetitive contraction of the flexor-pronator mass stresses the chondro-osseous origin, leading to apophysitis

This adolescent pitcher’s symptoms characterize Little Leaguer’s elbow. Little Leaguer’ elbow results from repetitive valgus stresses and tension overload of the medial structures.

Repetitive contraction of the flexor-pronator mass stresses the chondro-osseous origin at the medial epicondyle, leading to inflammation and subsequent apophysitis. Radiographic changes may range from normal to irregular ossification of the medial epicondylar apophysis, followed by accelerated growth, marked by apophyseal enlargement, separation, and eventually fragmentation.

This condition is secondary to overuse (typically throwing) and responds well to rest from the inciting activity. There is no evidence from the patient’s history or examination that he has an impingement syndrome, nor is there any indication of labral pathology on the MRI scan. The changes in the proximal humerus are classic for this condition and are not suggestive of a neoplastic process requiring biopsy for definitive diagnosis.

Little Leaguer’s shoulder is an overuse injury occuring in young baseball pitchers resulting in epiphysiolysis of the proximal humerus (a Salter Harris Type 1 injury).
Diagnosis is made with radiographs of the shoulder showing a widened proximal humerus physis in comparison to contralateral shoulder.
Treatment is cessation of throwing, followed by PT and progressive throwing program after sufficient rest.

M>F
11-16 years - skeletally immature overhead athletes
10% all shoulder pain in paediatrics is related to throwing.

Mechanism: repetitive torsional and distractive stresses at the physis (SHI injury) - affects hypertrophic zone of physis (weakest portion of the growth plate)

Pitching occurs in 3 phases:
- Late cocking: shoulder is maximally externally rotated, leading to extreme rotatory torque through the growth plate, approximately 400% greater than the fragile physeal cartilage can tolerate

  • Deceleration: opposing forces of forward arm motion and rotator cuff results in excessive eccentric physeal stress

Breaking pitches are implicated
Number of pitches is the most important factor

Presents with decreased pitch velocity and accuracy - causes diffuse arm and shoulder pain with throwing - worse in late cocking or deceleration phases.
Pain resolves with rest.

Examination: Point tenderness over lateral proximal humerus, at the level of the physis.
Pain reproduced with shoulder rotation, Glenohumeral internal rotation deficit.

27
Q

A 35-year-old man awoke following a night of heavy drinking with severe right shoulder pain and inability to raise his arm above his head and difficulty in external rotation. Only an AP XR is performed. He was treated with a sling for a diagnosis of rotator cuff tear. Six weeks later, he complains of continued pain and difficulty using the arm. Which of the following is the next best step in management?

Physical therapy for range of motion followed by rotator cuff and deltoid strengthening exercises
Axillary radiograph of the shoulder
EMG to evaluate the suprascapular and axillary nerves
Arthroscopic rotator cuff repair
Open subacromial decompression and latissimus dorsi transfer for massive cuff tear

A

Axillary radiograph of the shoulder

This question tests the concept that posterior shoulder dislocation is frequently missed due to inadequate imaging. Trauma shoulder radiographs (which include an AP, axillary, and scapular Y view) must be obtained in all suspected shoulder dislocations.

An Axillary radiograph is necessary to evaluate concentric reduction vs. dislocation of the shoulder. An example is provided in illustration A. Posterior shoulder dislocations can be easily be missed without the proper orthogonal views of the shoulder.

Posterior dislocations are more common following a seizure. The posteriorly dislocated shoulder is typically held in IR and most consistent finding is a mechanical block to ER caused by the anterior humeral head defect on the posterior aspect of the glenoid.

According to the reference by Robinson et al, good functional outcomes are associated with early detection and treatment of isolated posterior dislocations that are associated with a small osseous defect and are stable following closed reduction.

Posterior shoulder instability and dislocations are less common than anterior shoulder instability and dislocations, but are much more commonly missed.
Diagnosis is made radiographically in the setting of acute dislocations. Chronic instability can be diagnosed with presence of positive posterior instability provocative tests and confirmed with MRI studies showing posterior labral pathology.
Treatment may be nonoperative or operative depending on chronicity of symptoms, recurrence of instability, and the severity of labrum and/or glenoid defects.

Risk factors for posterior dislocation include bony abnormality (glenoid retroversion or hypoplasia) and ligamentous laxity.

Mechanism:
Trauma - posterior dislocation - usually significant trauma.
Microtrauma - posterior instability - can lead to labral tear, incomplete labral avulsion or erosion of the posterior labrum which may lead to gradual stretching of the capsule and patulous posterior capsule - usually insidious onset and presentation.
Seizures and electric shock - tetanic muscles pull the humeral head out

Biomechanical forces: flexed, adducted and IR rotated arm = high risk position

Static restraint: labrum deepens glenoid by 50%

Primary stabilisers of posterior shoulder:

  • Posterior band of IGHL: primary restraint in internal rotation
  • Subscapularis: primary dynamic restraint in external rotation and against posterior subluxation
  • Superior glenohumeral ligament and coracohumeral ligament: primary restraint to inferior translation of the adducted arm and to external rotation and primary static stabilizer to posterior subluxation with shoulder in flexion, adduction, and internal rotation.

Acute posterior dislocation -> limited external rotation, with the arm locked in an IR position. Pain on flexion, adduction and IR for posterior instability.

Provocative tests:

  • Jerk test
  • Kim test
  • Posterior stress test
  • Posterior load and shift test
28
Q

A 40-year-old semi-professional quarterback presents with non-specific shoulder pain and fatigue. He describes soreness at the base of his neck and over his deltoid and reports weakness with overhead activities. He has asymmetry of the scapulae on examination. Ultimately you decide to treat his condition with serratus anterior strengthening and stretching. However, what additional physical exam finding is most likely present?

Support of the inferior-medial scapular border during arm elevation decreases pain and improves range of motion
After the shoulder is placed in abduction and external rotation, it falls into internal rotation
The shoulder is placed in front of the body in internal rotation and hand presses on the abdomen, however the elbow falls back toward the body
With the patient supine, the shoulder is placed in abduction and external rotation – which causes discomfort
The patient’s arm is passively external rotated and abducted while his head turns to the contralateral extremity, which causes a decrease in the radial pulse.

A

Support of the inferior-medial scapular border during arm elevation decreases pain and improves range of motion

A common physical exam finding in medial scapular wigning is the scapular assistance test (SAT). On the SAT, as the patient elevates his arm, the examiner pressures an upward force on the inferior medial angle of the scapula. If this support alleviates some discomfort and the arc of motion improves, this is a positive test.

Medial scapular winging is a dysfunction in the scapular stabilizing muscles which results in excessive medializing scapular retraction and elevation. Medial winging is more common than lateral and is seen in younger athletic patients. It may be seen after a direct injury to the serratus anterior, injury to the long thoracic nerve, or in the setting of scapular dyskinesis from overuse. Therapy is the mainstay of treatment and the majority of patients will have a full return to function and resolution of winging within 2 years. Earlier surgical intervention is indicated when there is a known injury to the long thoracic nerve. Ultimately for recalcitrant cases, a split pectoralis major transfer can be considered.

Kibler et al reviewed scapular dyskinesis and its relation to shoulder injuries. The authors content that scapular dyskinesis can be found in association with many shoulder injuries. Generally, scapular wining and snapping can be managed with a rehabilitation protocol focusing on scapular stabilization. Those with associated primary shoulder conditions such as impingement, rotator cuff tendonitis, clavicle injuries, or instability should be evaluated for contaminant scapular dyskinesis.

Meininger et al. report that lesions of the long thoracic nerve and spinal accessory nerves are the most common cause of scapular winging, although numerous underlying etiologies have been described. They report that patients describe diffuse neck pain, shoulder girdle discomfort, upper back pain, and weakness with shoulder abduction and overhead activities. Most cases are treated nonsurgically.

Incorrect Answers :
After the shoulder is placed in abduction and external rotation, it falls into internal rotation: The horn blower’s sign is positive in those with teres minor pathology

The shoulder is placed in front of the body in internal rotation and hand presses on the abdomen, however the elbow falls back toward the body: The belly press sign is positive in the setting of subscapularis pathology

With the patient supine, the shoulder is placed in abduction and external rotation – which causes discomfort: The apprehension sign is positive in those with anterior shoulder instability

The patient’s arm is passively external rotated and abducted while his head turns to the contralateral extremity, which causes a decrease in the radial pulse.: Wright’s test is positive in those with thoracic outlet syndrome

29
Q

A 20-year-old female presents with recurrent anterior shoulder instability. Imaging is obtained and demonstrates a bony Bankart lesion involving 40% of the glenoid. A Laterjet procedure is planned for the patient. Which of the nerves is most at risk during the planned procedure?

Axillary Nerve
Musculocutaneous Nerve
Radial Nerve
Ulnar Nerve
Median Nerve
A

Musculocutaneous Nerve

The patient is planned to undergo a Laterjet procedure, which places the musculocutaneous nerve at highest risk for injury.

Patients with glenoid bone defects >20-25% have a high recurrence rate (>60%) after Bankart repair alone. Bone grafting is necessary to offer containment. Autograft options include coracoid transfer (such as the Latarjet procedure which extends the articular arc and creates a conjoined tendon sling), iliac crest bone grafting, and distal tibial bone grafting. When performing a Laterjet procedure, the musculocutaneous nerve is the most commonly injured nerve and occurs due to instrumentation around the conjoint tendon. The musculocutaneous nerve passes through the coracobrachialis muscle and descends between the biceps brachii and brachialis muscles, giving rise to the lateral antebrachial cutaneous nerve (LACN). Injury to the musculocutaneous nerve results in weakness of the biceps brachii muscle and a sensory deficit in the distribution of the LACN. The axillary nerve may also be injured during graft fixation in a Laterjet procedure, but its incidence is less common than musculocutaneous nerve injury.

Gupta et al. reviewed the complications of open and arthroscopic Latarjet procedures. Despite the promising results in addressing shoulder instability, they found that the rate of complications was between 15% and 30%. They report that the risk of neurovascular injury is reported as being between 1.4% and 10%, with the musculocutaneous nerve being the most commonly injured nerve due to instrumentation around the conjoint tendon.

Important to note that the Latarjet procedure results in consistent and clinically significant alterations in the anatomic relationships of the musculocutaneous and axillary nerves, which may make them vulnerable to injury during revision surgery.

Clavert et al. published a study on the relationships of the musculocutaneous nerve and the coracobrachialis during the Latarjet procedure. They found that injuries to the musculocutaneous nerve that occur during a Laterjet procedure were due to lengthening of the nerve and modification of the penetration angle of the nerve into the coracobrachialis. They also suggested that some motor nerve destined to the coracobrachialis might be damaged during the proximal medial release of the muscle after the detachment of the pectoralis minor muscle.

Incorrect Answers:
Answer 1: The axillary nerve can also be injured during a Latarjet procedure, especially during graft fixation, but its incidence is less common than musculocutaneous nerve injury.
Answers 2, 3, and 4: The radial nerve, ulnar nerve, and median nerve are less likely to be injured by the Latarjet procedure, due to their paths.

Older (>20 years old), recreational athletes with minor glenoid bone loss (<20% of the glenoid surface area) may be treated with soft tissue stabilization procedures using suture anchors. Goals of this procedure include tightening and repairing the torn ligament and labrum to the glenoid. Younger, contact sports athletes with large glenoid defect (>20%) may require bony augmentation type of procedures.

Defects larger than 25% of glenoid width should be managed with bony augmentation, with soft-tissue stabilization in smaller defects.

Following Bankart procedure risk factors for failure: age <=20, competitive participation in contact sports, shoulder hyperlaxity, Hill-Sachs on AP radiograph, glenoid bone loss of contour on AP radiograph.

The MOON Shoulder Group compared radiography, MRI and CT to determine the most reliable imaging modality for predicting bone loss. Three-dimensional CT, followed by regular CT were the most reliable and reproducible imaging modalities for predicting glenoid bone loss.

Remplissage procedure is indicated in setting of large Hill-Sachs lesions.

Glenoid augmentation procedures using coracoid transfer (Bristow-Latarjet) and tricortical iliac crest graft would be indicated in setting of larger glenoid bone loss.

Arthroscopic Bankart Repair +/- capsular plication:
- Relative implications: 1st time dislocation with Bankart on MRI in athlete <25 years, high demand athletes, recurrent dislocation/subluxation (>1 dislocation) following non-operative management <20-25% bone loss, remplissage augmentation with arthroscopic Bankart may be considered if Hills-Sachs “off-track”.
Technique: at least 3 anchor points should be used, paramount that labrum is fully mobilised prior to repair.
Equal outcomes to open repair with advantage of less pain and greater motion preservation.
Failure rates increased in patients with global hyper laxity, glenoid bone loss or too few fixation points.

Laterjet (coracoid transfer) or Bristow Procedure:

  • For chronic bony deficiencies with >20-25% glenoid deficiency (inverted pear deformity to glenoid).
  • Transfer of coracoid bone with attached conjoined tendon and CA ligament
  • Laterjet has the triple effect = bony (increases glenoid track), sling (conjoined tendon on top of subscap), capsule reconstruction (CA ligament)
  • Deltopectoral approach, split subscap
Consider autograft (tri-cortical Iliac crest or distal clavicle) or allograft (iliac crest or distal tibia) for bony deficiencies with >20-25% glenoid deficiency (inverted pear deformity to glenoid) and revision of failed latarjet. 
Can be performed arthroscopic or open. 

Remplissage and Bankart Repair:
- For engaging large (>25-40%) Hill-Sach or ‘off-track’ Hill-Sach lesion with <20-25% glenoid bone loss
- Technique: posterior capsule and infraspinatus tendon is sutured into the Hill-Sach lesion, may be performed with concomitant Bankart repair. By decreasing size of Hill-Sach, converts off-track lesion into on-track lesion.
When compared to latarjet at 2 years, replissage and Bankart has lower recurrent instability rates (1.4% vs 3.2%) despite greater bipolar bone loss.

Bone graft recon for Hill-Sachs Defect
- For engaging large (>40%) Hill-Sach lesion
Technique: allograft reconstruction, arthroplasty, rotational osteotomy

Consider tendon transfer for chronic irreparable subscapularis tears (lat dorsi or sternal head of pec major)

30
Q

The greatest stress on the medial ulnar collateral ligament of the elbow occurs during which phase of throwing?

Wind-up
Early cocking
Late cocking
Ball release
Follow through
A

Late cocking

The late cocking and early acceleration phase of the overhead throw causes the greatest amount of valgus stress to the elbow. During this phase, the forearm lags behind the upper arm and generates valgus stress while the elbow is primarily dependent on the anterior band of the UCL for stability. The windup phase is benign for the elbow. In early cocking, the rotator cuff and deltoid are active, not the elbow. Ball release is the culmination of cocking and acceleration, but the maximal joint forces have already occurred across the UCL due to the rapid combination of valgus and extension during late cocking and early acceleration. In deceleration, the elbow flexors are most active to prevent hyperextension.

The medial UCL is subjected to near-failure tensile stresses during the late cocking/early acceleration phase of throwing.

The medial ulnar collateral ligament, or medial collateral ligament of the elbow, is composed of three bundles:

  • an anterior bundle
  • a posterior bundle
  • a variable transverse oblique bundle.

The anterior bundle of the ulnar collateral ligament is the primary restraint to valgus force of the elbow from 30 to 120 degrees of flexion.

Biomechanical testing has shown that valgus forces as high as 64 N.m at the elbow during late cocking and early acceleration phases of throwing with compressive forces of 500 N at the lateral radiocapitellar articulation as the elbow moves from 110 to 20 degrees of flexion and velocities as high as 3000 deg/sec.

Mechanisms of injury MUCL:
- Acute trauma: often associated with elbow dislocations
- Overuse injuries:
+ Microtrauma from repetitive valgus stress leads to rupture of anterior band of MUCL
+ Baseball pitchers place significant valgus stress on the elbow in the late cocking and early acceleration phase of throwing
+ elbow valgus load increases with poor throwing mechanics and decreases with trunk-scapular kinesis, forearm pronation, dynamic flexor-pronator stabilization

  • Iatrogenic : excessive olecranon osteophyte resection places MCL at risk
31
Q

Which patient would expect to have the longest implant survivorship after undergoing total elbow arthroplasty (TEA)?

Rheumatoid arthritis with unconstrained TEA
Rheumatoid arthritis with semiconstrained TEA
Primary osteoarthritis with semiconstrained TEA
Primary osteoarthritis with unconstrained TEA
Acute distal humeral bicolumnar fracture with semiconstrained TEA

A

Rheumatoid arthritis with semiconstrained TEA

Total elbow arthroplasty (TEA) has the longest average implant survivorship in patients with rheumatoid arthritis, above those seen in TEA done for fractures, flail elbow, osteoarthritis, and post-traumatic arthritis. Implant options have traditionally shown best results with semiconstrained TEA designs.

The primary surgical management of the rheumatoid elbow is mainly synovectomy or total elbow arthroplasty (TEA). The indications for TEA include advanced disease of the elbow (grade III or IV), significant pain, and limitations in activities of daily living.

Contraindications include active or historical elbow infection, upper extremity paralysis, and refusal to abide by postoperative instructions. Implant options are traditionally classified as linked (semiconstrained) or unlinked - with improved stability and survivorship of semiconstrained designs.

Incorrect Answers:
Answers 1,3-5: Constrained implants have a higher rate of failure than unconstrained or semi-constrained implants. Validated databases have shown the longest survival in patients with RA compared to fractures, flail elbow, osteoarthritis, and post-traumatic arthritis.

Total Elbow Arthroplasty:

  • An increasingly used motion-preserving modality – primary indications include rheumatoid arthritis, post-traumatic arthritis and intra-articular distal humerus fracture in elderly with poor bone quality, or chronic instability.
  • Semi-contrained implants have the best longevity and optimal functional outcomes.

Rheumatoid arthritis: 10-20% patients will have arthritic elbow changes, consider TEA for Larsen stages 3-5 with functional loss, pain, instability. TEA for RA has longest survivorship when compared to other indications – most reliable with advanced, refractory RA. Pre-op C spine XR.
92% revision free survivorship at 10 years but high complication rate (14%): triceps avulsion, deep infection, periprosthetic fracture, aseptic loosening.

Primary OA: >65years old, mid arc pain with activity due to ulnotrochlear joint cartilage loss.
10 year implant survival 80-85%

Post-traumatic OA: 5 year survivorship – most achieve functional ROM and patient satisfaction but high complication rate (>30%), high re-operation rate (25%).

Fracture: physiologically elderly patient with acute complex, unreconstructable intra-articular distal humerus fracture or missed elbow fracture-dislocation with poor quality bone.
Gives faster recovery with more predictable outcomes vs fixation. Limit weight lifting to 2-4kg to avoid implant loosening.

Contra-indications:
Absolute: active joint infection (arthrodesis favoured), Charcot joint
Relative: poor neurologic control affected extremity, active patient <65 years, olecranon osteotomy – must be able to comply with post-op weight bearing restriction, low demand patient.

Pre-op: assess bone stock, ensure medial and lateral columns intact, assess canal diameter for implant design.

Implants:
- Unconstrained or unlinked components: requires competent collateral ligaments and soft tissue envelope, as well as good bone quality. Instability is commonest complication (5-10% dislocation). Precise component alignment is required. No proven superiority to semiconstrained/linked.
- Semi-constrained: ‘sloppy’ hinge allows for some varus-valgus and rotational laxity – this reduces the bone-cement interface stress and therefore incidence of component loosening.
o Best results of all the designs, early humeral loosening complication of designs without an anterior flange.
- Constrained: rigid hinged design, theoretically most stable design. Highest rates of loosening vs semiconstrained and unconstrained designs.

Design Features:
• Component stems (ulna and humerus) have improved fixation and reduced loosening
• Humeral component extracortical anterior flange resists posteriorly directed and rotational forces
• radial head not needed for stability in linked TEA designs
o radial head often debrided or resected in RA, due to mechanical symptoms or pain
• Semiconstrained most common
• Modern cement preparation and technique

Approaches: Triceps-reflection, triceps-splitting and triceps-sparing
• Triceps-reflecting (Bryan-Morrey): Triceps reflected from medial to lateral in continuity with the anconeous then re-attached to ulna with nonabsorbable suture through bone tunnels
o There can be associated weakness or loss of elbow extension due to the detachment of the triceps brachii insertion from the olecranon.
• Triceps tongue: raise fascial tongue from olecranon back proximally then release collateral ligaments proximally and distally. Can be used for fractures or TEA
• Triceps-splitting: triceps is longitudinally divided in continuity with forearm fascia over dorsal ulna. Triceps can also be split proximally with a V-shaped turndown of the tendon, leaving insertion onto olecranon intact - allows for extensor mechanism lengthening if needed.
• Triceps-sparing: triceps preserved intraoperatively, but exposure can be challenging, medial and lateral borders of triceps are mobilized. Best for using TEA to manage acute distal humerus fractures.
• Triceps “on”: direct midline, posterior incision, identify, release and protect the ulnar nerve, release the flexor-pronator mass and medial collateral ligament from medial epicondyle. Elevate the triceps off the posterior humerus towards the lateral intermuscular septum - release the common extensors and lateral collateral ligament complex and disarticulate the ulno-humeral joint

• Bone preparation
o humeral component: resect the olecranon fossa of distal humerus
 keep medial and lateral column intact
 broaching to appropriate sized component
o ulnar component: resect the olecranon tip of proximal ulna and the tip of coronoid to avoid impingement on anterior flange which will cause axial pistoning of ulna and loosening
o Broaching to appropriate sized component

  • humerus component: prepare a wedge-shaped piece of bone for placement behind the humeral flange (maintains component orientation relative to the posterior flat surface of the distal humerus)
  • ulnar component: orient the implant perpendicular to the dorsal flat surface of the olecranon

Complications:
- Aseptic loosening: radiographic 17%, clinical 6% - most common mode of failure for constrained
- Infection (8%): acute (<30days) -> aggressive serial irrigation and debridement and antibiotic bead placement. Staph epidermidis associated with persistent infection as encapsulating organism. Chronic: 2 stage or resection.
- Instability (7-19%) – most common mode of failure in semiconstrained.
- Brushing wear – commonest mode of failure for constrained – varus/valgus angle >10 deg is concerning.
- Wound healing, triceps insufficiency, ulnar neuropathy, bone loss
- Peri-prosthetic fracture: 5-30% TEAs. Classified by Mayo (O’Driscoll & Morrey)
o Type I: peri-articular fracture of humeral condyle or olecranon due to osteolysis around hinge components and distracting forces from muscle attachments. Undisplaced doesn’t require rigid fixatiob (immobilisation or soft tissue repair), displaced: ORIF with heavy non-absorbable sutures or TBW.
o Type II: Fracture along length humeral or ulna stem -1. Well fixed (ORIF component retention +/- strut allograft), 2. Loose implant, good bone stock, (revision arthroplasty – long stem +/- cerclage wires) 3. Loose implant, bone loss. (revision arthroplasty with allograft or resection arthroplasty).
o Type III: distal to prosthesis: well fixed implants -> immobilise humerus, ORIF ulna, loose implants treat as II2 fractures.

32
Q

A 44-year-old left-hand dominant carpenter experienced immediate left elbow pain after trying to stop a heavy object from falling two days ago. Physical exam shows full strength with wrist flexion, wrist extension, and pronation, but notable weakness with supination of the forearm. Sensory exam shows no deficits in the forearm or hand. There is a negative milking maneuver test and a positive hook test. Radiographs are shown unremarkable. What is the next most appropriate step in management?

Sling use as needed for comfort and progressive physical therapy
Allograft reconstruction of the distal biceps tendon
Ulnar collateral ligament reconstruction
Distal biceps tendon avulsion repair
Brachioradialis and ECRB avulsion repair

A

Distal biceps tendon avulsion repair

Distal biceps tendon avulsion repair is the most appropriate next step in management.

Distal biceps tendon ruptures occur most commonly in middle-aged men and usually involve the dominant extremity. The mechanism of injury is usually a single traumatic event with eccentric force on the flexed elbow.

Sutton et al. authored a Level 5 review of distal biceps tendon ruptures. They discuss that nonsurgical management of distal biceps tears is appropriate in the low-demand or medically ill patient. Surgical repair improves elbow flexion strength by 30% and supination strength by 40% compared to nonoperative management.

O’Driscoll et al. conducted a Level 2 study examining the accuracy of the hook test for distal biceps rupture diagnosis. They found that the hook test was abnormal in 33 of 33 (100%) patients with complete biceps avulsions, and intact in 12 of 12 (100%) with partial detachments.

Incorrect Answers:
Answer 1: Non-surgical management is not indicated in a healthy, middle-aged patient with a distal biceps tear of the dominant arm.
Answer 2: Allograft reconstruction may be needed in chronic cases.
Answer 3 & 5: UCL reconstruction or mobile wad tendon repair is not indicated in this patient. A positive milking maneuver would be consistent with a UCL injury.